Blog | Blueprint Prep

How to Make a Statement with Your ERAS Personal Statement

Med School Tutors

  • June 29, 2023
  • Reviewed by: Amy Rontal, MD

You’re guaranteed to write a better ERAS personal statement if you follow these 12 tips.

Dr. Leila Javidi, Taylor Purvis, and Dr. Brian Radvansky contributed to this article.

Starting your residency application can feel like an overwhelming task, especially when it comes to writing your ERAS personal statement. It’s not clear why essays of this nature are so intimidating—maybe it’s because not all medical students are well-versed in language arts, many of us dislike writing, or maybe just the thought of putting “who you are” onto paper brings to the surface some uncomfortable feelings of self-awareness (whoa—this just got intense!).

This is a joke or course, but to be honest, sometimes when we sit down to write our ERAS personal statement we immediately think things like, “I’m not that interesting,” or “I haven’t done anything cool in life, I’ve spent most of my time in school thus far.” And that is completely normal. The majority of us haven’t had those pivotal moments in life that shake the ground beneath us and form a new foundation for who we are, and that’s OK!

Your ERAS personal statement isn’t intended to be a best-selling memoir. It’s intended to add another dimension to the otherwise black-and-white application full of scores and grades. It is an opportunity to show program directors your personality, what motivates you, and what you’re looking for in a residency program.

While you’ve probably heard all of this before, we bet you have more specific questions about how to tackle the ERAS personal statement. All of us sure did! So, without further ado, h ere are answers to the 12 most important questions about medical residency personal statements.

12 Frequently-Asked Questions About the ERAS Personal Statement

residency application timeline

1. How big of a deal is my ERAS personal statement to program directors?

According to the 2020 NRMP program director survey , 78% of program directors cite the ERAS personal statement as an important factor in deciding which candidates to interview,  making it the fourth-highest ranked factor behind USMLE Step 1, USMLE Step 2, and letters of recommendation. So, it’s pretty important in the grand scheme of your application!

Now, from experience in talking to different program directors and mentors, it’s clear that the most important thing is that your ERAS personal statement is well organized, well written, with proper grammar, no red flags, and that it’s only one page single-spaced. The standard ERAS personal statement length is typically 500-800 words (roughly four paragraphs).

A personal statement typically isn’t the “maker” of your residency application—however, it can be a deal “breaker” if it doesn’t have those attributes. That said, if you have a memorable, well-written personal statement, program directors will mention it, and it will make you stand out as an applicant. If they are on the fence about whether or not to interview you, a personal statement could potentially be the deciding factor. So, it’s pretty important!

2. What are things I should include in my ERAS personal statement?

A good ERAS personal statement should include the following: 

A catchy introduction to grab the reader

There are different ways to go about doing this, but if you’re stuck, an effective way to grab the reader’s attention is to open with a patient vignette. An interesting case is sure to pique the curiosity of your reader and keep them engaged as they read. Preventing boredom is something to strive for, as your application is one of perhaps hundreds that they are reading.

Ultimately, though, remember this is a personal statement. After you reveal the diagnosis or outcome of the patient vignette, you need to let the reader know what the case meant to you! The point of relating the vignette is to reveal something about yourself, not just present an interesting story about a patient. 

An overview of your desirable qualities

When letting the reader know what your positive qualities are, it’s important to remember a basic rule of good writing: SHOW, don’t tell. For example, instead of saying you are compassionate, describe a story from your life that demonstrates your compassion.

Highlights from your life experience 

This includes jobs, extracurricular activities, and hobbies that would help you to be an ideal candidate for whichever residency you are applying to. Pro tip: DON’T REGURGITATE YOUR CV. This is your opportunity to tell people things that aren’t on your CV. Do you play chess in the park every Saturday, or have you traveled to some amazing places? Tell us about it!

You shouldn’t rehash your CV in your personal statement, but it is a great place to elaborate on activities listed on your CV. It can be used to explain why those activities are so important to you, how they have helped you grow as a person, and other things that don’t often shine through on the CV itself.

Proof of why you should be accepted 

The most important part of your statement is providing proof of why you should be accepted. Describe your strengths, but do not talk about things too generally. You should be able to back up everything you say. Give details and examples. Which doctors have you shadowed? What kind of research have you been involved in, and where was it published? Don’t just mention that you have volunteered, say the names of places you were at and what you were doing.

Why you are interested in your specialty

This doesn’t have to be a profound story, but it should be the truth!

What you are looking for in a residency program

Is a strong procedural curriculum important to you? Is the culture of the program more important? Try to mention things you know your programs of choice embody.

Address any red flags on your application

Did you do poorly on Step 1? Did you take a leave of absence for a long time? Best to just come out and talk about it without being defensive. Show how you have grown from the experience, rather than apologizing for it!

A cohesive closing statement

Sometimes the first and the last sentence of the statement are the hardest to come up with, but it’s worth your time to make it tidy, even if it isn’t profound.

3. What are things I shouldn’t include in my ERAS personal statement?

Controversial topics.

Stay away from extreme religious or political statements. It doesn’t mean you can’t say you are an active member of church, but don’t use this as an opportunity to discuss whether or not you are pro-choice. You never know who is going to be reading this, and anything too polarizing can be off-putting for some readers. 

Feelings of bitterness or negativity

Leave out any traces of bitterness, defensiveness, or anger about anything that has happened in your life. Everything must have a positive spin.  

Too much self-praise or too much modesty

Avoid talking about yourself in a glorifying manner, but don’t go too far the other way and come off as too modest.

Too many qualifiers

You don’t want to go overboard with the qualifiers, which are words such as “really,” “quite,” “very,” etc. In fact, in many cases, it’s better not to use them at all. 

“Flowery” language you wouldn’t use in real life

It’s a personal statement, not a creative writing assignment. Keep the language in your statement simple. You’re not going to score any points by using unnecessarily fancy words. Your goal is clear communication.

Also, don’t try to sound like a doctor. This is just another way of trying to impress the reader. You want the reader to like you based on the way you write, not be turned off because you are trying to impress them.

“Try to avoid using a lot of jargon and abbreviations,” advises Mary Dundas, educator at Academized. 

Exaggerations

Avoid talking hyperbolically about how passionate you are. As noted earlier, it’s better to show than tell so give examples of things you have done. Above all, keep the writing in your statement professional.

If you avoid these common mistakes, you’ll be way ahead of most applicants! 

4. How can I make my ERAS personal statement unique?

As evidenced by The Voice and American Idol , it is everyone’s impulse to divulge their “sob story” to help them stand out and garner sympathy from the audience. While it’s important to include stories that helped shape you as a person, it is very transparent and cliché to talk about that person you know who was struck by a medical tragedy, and how ever since you vowed to “save people.”

The best way to make your statement unique is to allow your personality to shine through. Use your words, your humor, and your depth to tell your story. Find a way to show yourself to your reader, and if you do this, your essay will be unique!

5. Should I have more than one ERAS personal statement to upload?

In short, absolutely have multiple personal statements to upload. Especially if you are applying to more than one specialty, it’s essential that you have several versions of your personal statement.

That doesn’t mean you have to write a whole new one, you just have to tailor it to fit that specialty. If you’re applying for a preliminary year, tailor your personal statement to explain how important you feel a solid foundation in medicine is for dermatology (or whichever specialty you are applying to) and what you’re looking for in a preliminary year.

Furthermore, I found that for the programs I really wanted to interview with, I would upload a tailored personal statement for that program saying something like, “I am seeking a family medicine residency position with ABC University program because of their dedication to XYZ.” Simply name-dropping their institution and noting the strength of their program demonstrates your attention to detail and interest in their institution. Even if you are an amazing applicant, if a program doesn’t feel you are interested in their specific program, they won’t interview you. It’s best to make sure you give those out-of-state programs some extra attention so they know you are willing to relocate for them!

Lastly, you should know that you can upload as many versions of your personal statement as you like onto ERAS, but be especially careful when uploading and make sure you apply the correct personal statement to each program! Triple-check your work! Pro Tip: Use your file names to help you stay organized. Pick a format and stick with it, such as “PS-JohnsHopkins,” “USCF-PS,” etc.

6. When should I start writing my ERAS personal statement?

The sooner the better, people. Get cracking now! You can even begin to think of ideas during your third year as you develop your interests in specific specialties. As ideas come to you, jot them into your phone so you don’t forget!

One of the best ways to begin writing your personal statement is to go over some questions about yourself. Ask yourself, who are you and what drives you forward? Think about the kinds of things that interest you and why you developed those interests. Maybe consider some mistakes you have made, how you learned from them, and how they have changed you. Or ask yourself, how do your interests and personality contribute to the goals you have set? 

Think about those kinds of questions and write down the answers. Reflect on them, put them away, and come back to them. Then, use them to form an outline—this will help you figure out all your points and what you want to say before you start writing. 

If you still feel like you just don’t know how to get started, give the five-point essay format a shot and see if it works for you. In short, you begin with a paragraph that is about four or five sentences long. The goal of this first paragraph is to grab a reader’s attention. Use the next three or four body paragraphs to talk about yourself. Try and have one of them focus on your clinical understanding, while another talks about service. Then end with a solid conclusion paragraph that mirrors your introduction, summarizes who you are, and ends by looking toward the future. 

7. Should I ask for any help with my ERAS personal statement?

Yes. Yes. A thousand times, YES! Absolutely ask for feedback on your personal statement. After getting your draft finished, show it to whoever will look at it—however, please remember to take everyone’s advice with a grain of salt and to strongly consider the source. It is absolutely essential to have your personal statement reviewed by an objective third party to ensure that the message you are trying to communicate is loud and clear. This means that you shouldn’t give it to a friend or family member who is going to placate you with a useless, “Yeah, looks great!”

Find a mentor, advisor, chief resident or attending, someone who is accustomed to reading ERAS personal statements, and get feedback from them. You can be certain that going through this step will only make your personal statement better. If you take their advice and don’t like how things are panning out, you can always revert back to an older draft.

But in just about every case, another set of eyes to give you big-picture feedback on what you’ve written will improve your piece. Do this early in the process, when you have gotten a simple draft together, so that you don’t present someone with an idea that you are married to, only to find out that it doesn’t come through clearly.

Be sure to ask other people what they think of your draft, but be careful about asking other students for help. Sometimes they get weird, and try to give you advice about making your statement more like theirs because they want to feel justified in their own efforts.

Finally, it should be mentioned that there are services out there that will “write your personal statement” for you. Aside from the obvious reasons why not to do this, you have to be really careful. Those services don’t know you, don’t know your voice, and oftentimes have very generic ways of putting these statements together.  Using a service to help polish your statement, though, is A-OK. Some you may find useful in that regard are ViaWriting , Writing Populist , StateofWriting , and SimpleGrad .

Lastly, you may consider working with a residency counselor who can help set your application apart with insider advice and ensure you optimize all elements of the residency application process. Our residency consultants are residents and attendings who have successfully guided hundreds of students from residency applications through the Match!

Typical residency consulting work consists of:

residency consulting

Not sure if a residency consultant is the right fit for you? Take this quiz to see if you would benefit from some extra guidance during the residency application process!

8. Where can I find examples of ERAS personal statements to inspire me?

Every good writer learned how to write by reading the works of other people. This includes personal statements! Very often your career offices from your undergraduate studies will have examples of personal statements that can serve as inspiration for your own masterpiece. You can also ask older classmates and recent graduates if they would feel comfortable sharing their personal statements with you. 

Remember, too, that inspiration can come from nontraditional sources. Try reading poetry or a novel before sitting down to write your statement. You might be surprised by how it helps to get your creative juices flowing!

9. Is it better to cover all of my experiences, or focus on a few in particular?

It’s better to focus on several key experiences rather than provide a broad overview of your life up to the present time. Your resume will fill in any gaps for your reader. The point of the personal statement is to spend a few paragraphs reflecting on one or two themes that define who you are as a person. Stay focused, and go deep!

10. How much should I share about my career goals in my ERAS personal statement?

Remember, the majority of training programs you will be applying to are academic medical centers. For those programs in particular, make sure to emphasize why an academic environment is a good fit for you. This does not have to mean research! Perhaps you like the idea of becoming a clinician educator and want to be at XYZ program for the opportunity to teach medical students. 

Likewise, if you are applying to a program at a community hospital, make sure to reflect on how your career goals are suited for that environment. Maybe private practice is on your radar, or you want to practice in a hospital that is more close-knit than a large academic center.

Whatever the case, try to make your stated career goals align with the orientation of the program you’re applying to. In reality, you may have no idea what direction you want your career to go in. But for a personal statement, try to commit to one general theme if possible.

11. What about my personal statements for preliminary or transitional year programs?

For applicants who are also applying to preliminary or transitional year programs, it can seem daunting to tailor your personal statement to a position that isn’t part of your ultimate specialty. But don’t worry—preliminary and transitional year programs still want to know who you are as a person and why you’re interested in anesthesiology, dermatology, or whatever advanced specialty you’re aiming for. You don’t need to change your personal statement as much as you may think!

The goal of a personal statement for these one-year programs is not to convince the reader that you suddenly love internal medicine despite going into radiology. The reader knows this is a temporary stopping place for you. Instead, emphasize the traits that make you YOU and will enhance their hospital!

12. What if I’m interested in a non-traditional path after residency?

Some of you may be thinking of alternative career paths after residency such as consulting or pharmaceutical work. It’s probably best to leave those specific goals out of your ERAS personal statement and allow readers to assume that you want to continue in clinical medicine after graduating from residency. You might want to instead phrase it as something you want to be incorporated into your clinical career, but not something you would leave medicine for, even if that’s what you have in mind!

Remember, you are under no obligation to share your every thought and desire in a personal statement! These statements are being read by reviewers who dedicated their lives to education and clinical medicine, so keep that in mind.

Further Reading

Keep these tips in mind as you write your ERAS personal statement, and you’ll be way ahead of the other applicants. If you start to get stressed out, remember, you have an amazing story to tell, and we are here to help tease that story out from the confines of your brain! For more help, reach out to one of our residency advisors .

Looking for more help during the residency application process? We’ve got you covered with more (free!) content written by Blueprint tutors:

  • How to Get Standout Letters of Recommendation for Your Residency Application
  • How to Maximize Your Chances of Matching With Your Dream Residency
  • What’s It Like Working With a Medical Residency Consultant?
  • Residency Interview Tips & Tricks: The Ultimate Guide
  • Dual Applying for Residency: Is It Right For Me?

CTA Logo

Related Posts

Here’s what you need to know about the ERAS application 2025 updates.

Everything to Know About the ERAS Application 2025 Updates

  • July 18, 2024

Here’s some residency interview questions you’ll want to be ready for.

The Most Common Medical Residency Interview Questions

  • May 17, 2024

Use this residency application timeline to organize your efforts and stay on top of everything.

Navigating the ERAS Residency Application Timeline: The Ultimate Guide

  • April 26, 2024

MedEdits Logo

The Residency Personal Statement (2024/2025): The Insider’s Guide (with Examples)

Residency Match Personal Statement

A physician and  former residency program director  explains how to write your residency personal statement to match in to your top-choice residency program in 2025.

Read example residency personal statements and suggested outlines., introduction.

We have been working with residency applicants who successfully match into the programs and specialities of choice for more than 15 years and a key part of that success, is writing a compelling residency personal statement.

Having worked with so many applicants, we know you will get differing advice depending on who you ask. The key to our applicants’ success is that we understand how to write a residency personal statement that has broad appeal and will impress all types of readers.

The residency personal statement allows residency program directors and associate directors the chance to get a sense of who you are and your commitment to your chosen specialty. 

As a former program director who understands how residency personal statements are reviewed, what “stands out,” and, most importantly, what will earn you interview invitations, the information below will help you write a residency personal statement to match!

It is imperative to make sure you get the most accurate guidance possible with regards to your residency personal statement content and optimal residency personal statement length (up to one page).

Want more personalized suggestions?  Sign up for a FREE residency personal statement consultation .

Table of Contents

Goals for Writing Your 2025 Residency Personal Statement

Above all else, your residency personal statement offers the opportunity to show your interest in your  chosen specialty  when applying to  residency  to illustrate you are a good fit.

The more details you offer about why you are interested in the specialty and how your med school rotations,  accomplishments  and experiences have reinforced this interest, the stronger your personal statement will be, the more it will appeal to selection committees and the better you will do in the match process.

I encourage applicants to offer as much “evidence” as possible to “show” rather than “tell” what  qualities, characteristics and interests  they have. “Telling” a reader, for example, that you are compassionate and hard working means nothing. Instead, you must “show” that you embody these qualities based on your experiences in health care and the patients for whom you have cared.

The residency personal statement also offers the opportunity to write about who you are as a person to convey some details about your background, influences, and interests outside of your given specialty.

The Importance of a Balanced Residency Personal Statement

The key when writing your residency personal statement is to ensure that it is well-balanced so it appeals to a large group of people who might read your ERAS residency application.

However, it is important to understand that every program director and  faculty member  has his or her own idea of what he would like to read in a personal statement. As an applicant, you must go into this process understanding that you cannot please everyone, or a specific program, and your personal statement should therefore have the broadest appeal possible.

For example, some  program directors  would rather hear about your personal interests and curiosities and get to know who you are rather than have you focus on the specialty in which you are interested.

At MedEdits, we suggest taking a “middle of the road” approach; include some details about who you are but also focus on the specialty itself. In this way, you will make more traditional reviewers who want to hear about your interest in the specialty happy while also satisfying those who would rather learn about you as a person.

Above all, be authentic and true to yourself when writing your statement. This always leads to the best results! Read on to learn more about how to write a winning personal statement.

About MedEdits

Getting into a residency has never been more competitive. Founded by a former associate program director, the experts at MedEdits will make your residency personal statement shine. We’ve worked with more than 5,000 students and 94% have been matched to one of their top-choice programs.

Need Help With Your Residency Personal Statement?

Schedule a Free 15 Minute Consultation with a MedEdits expert.

Residency Personal Statement Outline & Structure

Residency applicants often do well when given outlines or templates to follow, so, we will offer that, but, it is important to realize that many applicants deviate from these rigid rules. One very typical outline that serves applicants quite well in the  residency admissions process  is:

  • Compose a catchy introduction. Your intro can be related to your  interest in the specialty  to which you are applying, about a hobby or personal experience, or about your background. Regardless of the topic you choose, you want to tell a story and start with something that will interest your reader and engage him.
  • The next two to four paragraphs comprise the body of your personal statement. We encourage applicants to write about any significant experiences they have had related to their desired specialty and/or future goals. This would include information about rotations, electives, and sub internships related to the specialty, volunteer and research experiences and even significant outside interests.
  • Finally, you want to conclude your essay. In your conclusion, write about what you seek in a residency program, what you will bring to a residency program, and, if you have any idea of your future career goals, write about those as well. Your conclusion is also where you can tailor a personal statement to a specific geographic area of interest or type of program (rural, urban, community).

Residency Personal Statement Length & Residency Personal Statement Word Limit

The allowed ERAS residency personal statement length is 28,000 characters which equates to about five pages!

We have been hearing from more and more applicants that the personal statement should not exceed  one page  when typed in to the  ERAS application . Because of this overwhelming trend, we are supporting this guidance unless you have  extenuating circumstances  that require your personal statement be longer.

Our recommendation is that your residency  personal statement be a maximum of 5300 characters with spaces.

ERAS Residency Personal Statement Checklist

  • Ensure your personal statement flows well

The best personal statements are easy to read, don’t make the reader think too much, and make your path and interests seem logical. Rarely does a personal statement have a theme. Also try to have each paragraph transition to the next seamlessly.

2. Your personal statement should be about you!

Your personal statement should be about you and no one else. Focus on your interests, your accomplishments and your path. This is your opportunity to be forthcoming about your  achievements  – by writing in detail about what you have done.

3. Be sure your personal statement clearly outlines your interest in the specialty.

Since the reader wants to be convinced of your understanding of, experience in, and curiosity about the specialty to which you are applying, be sure you highlight what you have done to explore your interest as well as your insights and observations about the specialty to show your understanding of it.

4. Make it human.

Again, your personal statement should be about you! The reader wants to know who you are, where you are from, what your interests are and who you are outside of medicine. Therefore, try to include those details about your background that are intriguing or important to you.

5. Express your interest in the specialty.

The reader fundamentally wants to know why you are pursuing the specialty. The more details you offer the more convincing you are about your commitment and your understanding of the specialty. Be sure to include details that might seem obvious. For example, in  emergency medicine  you must like acute care, but try to include more nuanced details about your interest, too. What aspects of the diagnoses and pathologies involved do you enjoy? What do you value about the actual work you will do? How do you feel about the patients for whom you will care?

6. The start and evolution of your interest.

Readers want to know how and when you became interested in your specialty. Was this before medical school? During medical school? What have you done to pursue and nurture your interest in the specialty?

7. What you have done to learn more about the specialty.

You should explain what you have done to pursue your interest. What rotations have you done or have planned? What research, scholarly work or community service activities have you pursued to further your interest?

8. Where you see yourself in the future – if you know!

Without going into too much detail, write about the type of setting in which you see yourself in the future. Do you hope to also participate in research, teaching, public health work or community outreach as a part of your career? What are your future goals? Since many programs typically train a certain type of physician, it is important that your goals are aligned with the programs to which you are applying.

9. What do you bring to the specialty?

You should try to identify what you can bring to the program and the specialty to which you are applying as a whole. For example, are you applying to family medicine and have a distinct interest in public health? Are you applying for  internal medicine  and do you have demonstrated expertise in information technology and hope to improve electronic medical records? Do you have extensive research or teaching experience, and do you hope to continue to pursue these interests in the future? Have you developed a commitment to global health, and do you hope to continue making contributions abroad? Programs have a societal obligation to select residents who will make valuable contributions in the future, so the more ambitions you have the more desirable a candidate you will be.

10. What type of program you hope to join?

Do you hope to be part of a community or university-based program? What are you seeking in a residency program? Programs are looking for residents who will be the right “fit” so offering an idea of what you are seeking in a program will help them determine if your values and goals mesh with those of the program.

11. Who you are outside of the hospital?

Try to bring in some personal elements about who you are. You can do this in a few ways. If you have any outside interests or accomplishments that complement your interest in your specialty, such as extracurricular work, global work, teaching or volunteer efforts, write about them in detail, and, in doing so, show the reader a different dimension of your personality. Or, consider opening your statement by writing about an experience related to your hobbies or outside interests. Write about this in the form of an introductory vignette. I suggest taking this nontraditional approach only if you are a talented writer and can somehow relate your outside interest to the specialty you are pursuing, however. An interest in the arts can lend itself to dermatology, plastic surgery or ophthalmology, for example. Or, an interest in technology could relate to  radiology .

12. Any personal challenges?

Also explain any obstacles you have overcome: Were you the first in your family to graduate from college? Were you an immigrant? Did you have limited financial resources and work through college? Many applicants tend to shy away from the very things that make them impressive because they are afraid of appearing to be looking for sympathy. As long as you explain how you have overcome adversity in a positive or creative way, your experience will be viewed as the tremendous accomplishment that it is. The personal statement should explain any unusual or distinctive aspects of your background.

Common ERAS Residency Personal Statement Mistakes

Do not tell your entire life story or write a statement focused on your childhood or undergraduate career. 

Do not write about why you wanted to be a doctor. This is old news. From the reviewers perspective, you already are a doctor!

Do not write a personal statement focused on one hobby or begin with your birth. Some background information might be useful if it offers context to your choices and path, but your residency personal statement should be focused on the present and what you have done to pursue your interest in the specialty to which you are applying.

Do not preach. The reader understands what it means to practice his specialty and does not need you to tell him. Don’t write, for example: Internal medicine requires that a physician be knowledgeable, kind and compassionate. The reader wants to know about you!

Do not put down other specialties. You don’t need to convince anyone of your interest by writing something negative about other specialties. Doing so just makes you look bad. If you switched residencies or interests, you can explain what else you were seeking and what you found in the specialty of your choice that interests you.

Do not embellish. Program directors are pretty good at sniffing out inconsistencies and dishonesty. Always tell the truth and be honest and authentic. 

Do not plagiarize. While this seems obvious to most people, every year people copy personal statements they find online or hire companies that use stock phrases and statement to compose statements for applicants. Don’t do it!

Do not write about sensitive topics. Even if you were in a relationship that ended and resulted in a  poor USMLE score , this is not a topic for a personal statement. In general, it is best to avoid discussing relationships, politics, ethical issues and religion.

Do not boast. Any hint of arrogance or self-righteousness may result in getting rejected. There is a fine line between confidence and self promotion. Some people make the mistake of over-selling themselves or writing about all of their fantastic qualities and characteristics. Rarely do readers view such personal statements favorably.

Do not write an overly creative piece. A residency personal statement should be professional. This work is equivalent to a job application. Don’t get too creative; stay focused.

Writing ERAS Residency Personal Statements For Multiple Specialties

An increasing number of applicants are applying to more than one specialty in medicine especially if the first choice specialty is very competitive. If you are applying to more than one specialty, even if there is disciplinary overlap between the two (for example family medicine and pediatrics), we advise you write a distinct specialty for each. Remember that a physician who practices the specialty you hope to join will most likely be reviewing your statement. He or she will definitely be able to determine if the personal statement illustrates a true understanding of the specialty. If you try to recycle an entire personal statement or parts of a personal statement for two specialties, there is a high likelihood the personal statement will communicate that you aren’t sincerely interested in that specialty or that you don’t really understand what the specialty is about.

Writing About Red Flags in your ERAS Personal Statement

The personal statement is also the place to explain any red flags in your application, such as gaps in time or a leave of absence. When addressing any red flags, explain what happened succinctly. Be honest, don’t make excuses, and don’t dwell on the topic. Whenever possible, write about how you have matured or grown from the adversity or what you may have learned and how this benefits you.

If you have left a program or had a break in your medical education, you will also have the chance to explain this in your  ERAS application . You should also write about this topic in your personal statement only if you have more to explain, however. 

If you have failed a Step exam or one course in medical school, this likely isn’t something to address in the personal statement. However, you should be prepared to discuss any failure during an interview. By the same token, it is best not to address one low grade or poor attending evaluation in your statement. 

Have you taken a circuitous path to medicine? If so you might address why you made these choices and what you found so interesting about medicine that was lacking in your former career.

Residency Personal Statement Example

Below are two great examples of residency personal statements that earned the applicants who wrote them numerous interviews and first choice matches. As you will see, these two applicants took very different approaches when writing the personal statement yet wrote equally persuasive and “successful” personal statements.

Residency Personal Statement Example, Analysis, and Outline: The Traditional Approach

Suggested outline:.

  • Introduction: Catchy Story
  • Paragraph 2: Background Information and how Interest Started
  • Paragraph 3: Write about what you did to explore your interest
  • Paragraph 4: Second paragraph about your experiences related to your specialty
  • Conclusion: Wrap it up. Write something about your future goals.

Below is an example of the traditional approach:

Why It’s Great

This is a great personal statement because it clearly conveys the applicant’s interest in, and understanding of, obstetrics and gynecology (OB/GYN) and what the applicant has done to pursue that interest. Not only does this applicant have a long-standing interest in OB/GYN, but, she conveys that she has experienced the specialty in different settings and understands the diverse nature of the specialty. She also includes information about her hobbies and interests and writes about her exploration of OB/GYN outside of the clinical arena. An added bonus is that the applicant writes well and uses descriptive language making her statement interesting and fun to read.

Residency Personal Statement Example, Analysis, and Outline: The Outside Interests Approach

Many mentors advise applicants to tell the reader something about them that is unrelated to medicine or the specialty they are pursuing. This is a fine idea, but be sure your personal statement also includes some details about your interest in your specialty if you decide to move in this direction.

Suggested Outline:

  • Introduction: Write a Catchy Introduction. Be creative! Think outside the box.
  • Paragraph 2:Elaborate on your introduction offering more details
  • Paragraph 3: Write about your specialty choice and what appeals to you.
  • Paragraph 4: Write more about your explorations in medical school.
  • Concluding paragraph(s): Write about your future goals, the type of program you hope to join and consider looping back to your introduction.

The landscape before me was lush and magical. We had been hiking for hours and had found a great spot to set up camp. As I was unloading my backpack and helping to pitch the tent, I saw a scene I knew I had to capture. I quickly grabbed my carefully packed Leica before the magnificent sunset disappeared. Trying to get the perfect exposure, I somehow managed to capture this image so accurately that it reflected the beauty of what was before us high in the mountains of Utah, so far away from the hustle and bustle of New York City where we attended medical school.

This is a really intriguing personal statement because the author writes about his outside interests in a compelling way that makes him instinctively likable. He then goes on to explain what he enjoys about surgery and what he has done to pursue that interest. As you can see, this applicant writes less about his specialty (surgery) than the applicant in statement #1 did, but, he still convinces the reader of his understanding of, and commitment to, surgery. In this statement, the reader gains a much broader understanding of who the applicant is as a person and what he likes to do in his free time.

Final Thoughts

Writing your residency personal statement should be about telling your story in your own voice and style. You want to highlight your interest in the specialty for which you are applying while also conveying some ideas about who you are as a person to keep your reader engaged in learning about you as a person.

Residency Personal Statement Consulting Services

MedEdits Medical Admissions offers comprehensive guidance and document review services for residency applicants to every specialty in medicine. With more than twenty years of experience in residency admissions and founded by a former residency admissions officer and physician, MedEdits understands what program directors want to read and can help you decide what aspects of your background to focus on in your residency personal statement to earn the most interviews possible.

Sample Residency Personal Statement Page 1

Residency Related Articles and Guidance

  • Residency Match Statistics
  • Residency Personal Statement
  • Residency Match: How It Works & How To Get Matched
  • How to write a residency interview thank you letter.
  • What Outfit To Wear To Your Residency Interview
  • Medical Residency Timeline & Length
  • Medical Residency Salary By Specialty
  • How To Master MyERAS, The Medical Residency Interview, and Common Residency Interview Questions
  • Master the ERAS (Electronic Residency Application Service) & ERAS Timeline
  • Residency Letters Of Recommendation (with ERAS Samples)
  • Residency Letter of Intent
  • How to Write a Residency Letter of Intent
  • Residency Love Letters
  • Residency Match Success: Lessons Learned

Residency Specialty Articles

  • Family Medicine Residency Match: Beat more than 4400 Applications
  • Pediatrics Residency Match: Beat more than 3000 Applicaitons
  • Internal Medicine Residency Match: Beat more than 10,000 Applications
  • General Surgery Residency Match: BEAT more than 1900 Applications
  • Emergency Medicine Residency Match: BEAT more than 2600 Applications
  • Anesthesiology Residency Match: BEAT more than 2,000 Applicants

MedEdits Medical Admissions Founder and Chairwoman, Jessica Freedman, MD

JESSICA FREEDMAN, M.D. , a former medical school and residency admissions officer at the  Icahn School of Medicine at Mount Sinai , is the founder and chair of MedEdits Medical Admissions and author of three top-selling books about the medical admissions process that you can find on  Amazon .

  • Website Disclaimer
  • Terms and Conditions
  • MedEdits Privacy Policy

personalized personal statement eras reddit

  • Residency Application

Residency Personal Statement Examples from Matched Residents

Residency personal statement examples

Use these residency personal statement examples as a reference as you are working on  preparing you residency applications . The following are printed with permission from our own past successful students who worked with us as part of our  residency application review  programs. If you are having trouble getting started, you are not alone. Many students find that the personal statement can be one of the most challenging components of the  ERAS  or  CaRMS  residency applications. However, your personal statement can make or break your application. Get started on the right track by following the guidelines outlined for you below reviewing the emergency medicine residency personal statement example , pediatrics personal statement example , cardiology personal statement example, and more..  

This blog will outline what types of things to include in your residency personal statement. It will also give you 10 examples of personal statements from 10 different specialties written by actual students who matched into those fields. Reviewing personal statement examples is also good essay writing practice if you decide to write a residency letter of intent . Many of the same principles you apply to the personal statement can be applied to other application materials as well, so consider this review comprehensive. Believe it or not, personal statements also entail a great deal of self-reflection, which means they also function as a great review for residency interview questions , like the “tell me about yourself” residency interview question .

>> Want us to help you get accepted? Schedule a free initial consultation here <<

Listen to the blog!

Article Contents 39 min read

Residency personal statement example #1: family medicine.

During the pre-clerkship years of study in medical school, I enjoyed learning about the many specialties within medicine and actively considered pursuing several of them. I was drawn to the complex pharmacology of the drugs used by anesthesiologists, the acuity of care faced by emergency medicine physicians and the complicated medical issues of patients cared for by internal medicine specialists. I also found myself interested in psychiatrists’ thorough history-taking and the technical skills in performing procedures exhibited by surgeons. It started becoming clear to me that I was interested in many different areas of medicine. I began realizing that I wanted a career that combined the many things I enjoyed in different specialties. A family physician has the flexibility to practice all of these facets of medicine. As clerkship drew nearer, I knew I wanted to gain more clinical experience in family medicine to see if it would be a good fit for me.

My clinical experiences in family medicine were fantastic. I worked with family physicians and family medicine residents not only during my core family medicine rotation and family medicine electives, but also during my psychiatry, surgery, anesthesiology, and pediatrics rotations. These clinical experiences confirmed my belief that family medicine is a diverse and exciting specialty; family physicians, while maintaining a broad base of medical knowledge, can tailor their practices to the needs of their communities and to their own interests and areas of expertise. During my family medicine rotation and electives, I also found myself greatly enjoying my encounters with patients. I enjoy hearing patients’ stories and sorting through their many medical and psychosocial issues. I am also naturally a fastidious person. Being a thorough history-taker and a meticulous recorder of details helps me in formulating a complete story about a patient. My joy in interacting with patients and my attention to detail allow me to appreciate patients as people, not just as disorders or diseases. I am both interested in learning about and have a certain affinity for, family medicine clinical experiences; pursuing a career in this specialty is an obvious choice for me.

The versatility and diversity of family practice initially drew my interest but the wonderful encounters I had with family physicians solidified my desire to pursue a career in this specialty. These family physicians have not only been skilled and knowledgeable clinicians but also, variously, dedicated teachers, researchers, and administrators. They were committed to improving their clinical skills by attending continuing education lectures and courses. They practiced patient-centered care and were knowledgeable about community resources that may help their patients. They worked cooperatively with other health-care professionals to improve patient care. Importantly, these physicians have also been friendly and approachable towards both learners and patients. The family physicians I have worked with also strive toward a healthy work-life balance; all of them seemed to have many interests and hobbies outside of their professions. These clinicians demonstrated to me what being a family physician involves: practicing both the science and art of medicine, advocating for patients, guiding patients through the health-care system, being committed to improving clinical knowledge and, importantly, maintaining one’s own health and happiness.

Being sure of the specialty I want to pursue is the first step in my career. There are many learning opportunities ahead. [Name of the program]’s family medicine residency program is attractive in so many ways: the protected academic days, the opportunity to participate in research and, most importantly, the clinical curriculum, all appeal to me. I believe the solid foundation of family medicine experience, as well as the exposure to other specialties, alongside the opportunities to build the skills necessary for life-long learning through the academic experiences and research, make this an ideal program for me. On a personal note, I grew up in [hometown] and did my undergraduate studies at [name of university]; I would be thrilled to return to my hometown and a university already familiar to me. My career goals after finishing my residency include having a community-based, urban family practice and being actively involved in teaching residents and medical students. I am also open to being involved in research and administration. Career goals, however, may change as I progress through my training. I am excited to begin the next stage of medical training and begin my residency in family medicine!

1. Emphasis on why the applicant wants to enter that specific specialty

This family medicine personal statement example does a great job of explaining why the applicant wants to enter that specific specialty. Their interest is clearly stated and the decision to enter the field is well explained. The author does an excellent job of talking up the specialty and stating what they like about the field based on their clinical experience. For your residency personal statement, you want to highlight any influential moment you had during these experiences. If you had a certain “aha” moment, you might mention this. If demonstrating this commitment is difficult for you, you can always find a reputable ERAS application review service .

2. Intentions are clear

Clearly stating your intentions and using the program's name makes your statement personal and stand out. It shows that you pay attention to details and that your goals and passion align with what the program offers. Use strong, precise language when you are writing. You only have about 800 words, so state your intentions and keep your story clear.

3. Personal connection is established

This particular applicant has a personal connection to the city in which the residency would take place. This won’t be true for every applicant, but if it is, be sure to make room to mention it as long as it fits with your personal narrative. In this example, the applicant also ties this in with one of their goals: having a community-based, urban family practice. In your personal statement, you should merge these elements together for a more cohesive essay.

What to Include in Your Personal Statement

Most residency programs, whether through  ERAS  (US-based) or  CaRMS  (Canada-based) require applicants to submit a personal statement or letter. Some programs will include specific instructions for what they wish you to talk about, while others will not give you a topic. When you’re doing your research for residency programs you want to apply for, you should also take a look at the selection criteria. Each school will have its own rubric that they use to evaluate candidates, so it’s a good idea to review these before you start working on your personal statement. Here is an example of some information stated by McMaster University regarding their residency selection criteria:

“Programs may consider a range of criteria in making their selection decisions for interviews including but not limited to: Medical School Performance Report (MSPR), scores on standardized tests, interest in and aptitude for the discipline, reference letter, experience in research or other scholarly activities, extracurricular activities, and personal qualities.”

ERAS, as well as most CaRMS programs, ask that your statement be within a one-page limit, about 750-850 words. Please check the specific program requirements through the ERAS or CaRMS websites.

The experiences in your  residency CV  can be used to help you indicate why you are applying to a particular program and how you came to that decision.

1. Introduction

Typically, your residency personal statement will have three to five paragraphs, which you will use to divide the introduction, body, and conclusion. The personal statement is a formal essay, so you must adhere to the proper structure. The introduction is for you to capture the attention of the reader; for this, you will need a strong hook or opening statement. Feel free to get creative with this. The remainder of your introduction should focus on what drew you to the specialty and how your background experiences informed your decision to apply to the school and program. Your introduction should also contain a thesis statement that allows you to connect your personal background with your suitability for the program, school, and a career in medicine (in this exact specialty).

2. Body (or middle)

The body of the essay is for you to expand on a few critical experiences that made you the excellent, qualified candidate you are today. A good strategy for the body paragraph(s) is to talk about relevant clinical rotation experiences; so for example, if you’re applying to a psychiatry residency, you can talk about a specific patient experience that solidified your decision to pursue this specialty, or an experience that sticks out in your memory. This will be similar to your answer to the interesting case residency interview question . Your goal should be to use these experiences to address your specific interests, goals, and what makes you a good fit for the program. Do some research into the program format, the patient population you will be working with, and the clinical environment. This will help you connect your experiences with what the school/program offers.

3. Conclusion

You might be thinking that once you’ve written a strong introduction and body, the conclusion will be simple. However, this isn’t necessarily the case. You need to use the space in your conclusion to tie everything together and show enthusiasm for the program and for your future career. You can revisit a few key points here to highlight them once again and to relate them to what you’re hoping to gain from the forthcoming training experience. Show passion, determination, and consistency throughout your letter and tie up any loose ends in the conclusion. Some applicants will use this part of the letter to mention a specific goal they want to achieve in residency, such as working with specific faculty members or research plans. You may also mention aspirations to complete a fellowship or what you want your future practice to look like.

Here's why "show, don't tell" is the most important tip for any personal statement:

Questions to Ask Yourself to Help You Brainstorm Ideas

  • What makes you right for this specialty?
  • What experiences drew you to this specialty?
  • What appeals to you about this specific program?
  • Do you have any experiences working in the city of the program you’re applying to?
  • How will your residency training help you achieve your goals?
  • What are some of your personal strengths that will allow you to contribute to the program?
  • What evidence do you have that you possess those strengths?
  • Do you have any research/publications that align with the research the school is doing?
  • Do you have any gaps in your medical education or evaluations that you would like to address?
  • What’s something you think the program director should know that isn’t obvious from your application materials?

  Growing up the first-born daughter of a hard-working Saskatchewan cattle farmer and hairdresser, medicine was never a consideration. In a small town, I could easily see how too much free time got many of my peers in trouble. From grade 8-12 I devoted myself to sports, playing high school, club and provincial beach volleyball, weeknights and weekends year round. Despite my small stature and lack of innate abilities, with determination and persistence, I overcame these obstacles. At the end of my grade 11 year, I received an athletic scholarship and chose to pursue business administration and athletics.

After the first six months, it became apparent that I was not going to attain my full potential in education at [university name}. Despite my parent’s reservations, I left and enrolled at a [university name] for the next semester. This university was much more challenging as I was now balancing my educational and financial responsibilities by working evenings and weekends managing a number of part-time jobs. With little direction as to what degree I wanted to pursue, I happened to enroll in anatomy and physiology. This was the first time I became really excited about my future prospects and began actively considering a career in medicine.

The first time I applied to medicine, I was rejected. Despite my initial devastation, in hindsight, it was a great opportunity for myself to reflect on my own motivations for medicine and work as a laboratory technician at a potash mine in my hometown. I gained additional life experience, spent time with my family and was able to help financially support my husband’s pursuit of education after he had so selflessly supported me for many years.

My first exposure to anesthesia was in my first year of medical school with [Dr. name here] as my mentor in clinical reasoning. I was again, intrigued by the anatomy and physiology with the interlacing of pharmacology. I remained open to all specialties, however, after summer early exposures, research, and clerkship it became clear to me that anesthesia is where I felt the most fulfilled and motivated.

In a way, anesthesia was reminiscent of the competitive volleyball I had played years prior. I was again a part of a team in the operating room with a common goal. Similarly, our countless years of education and practice had brought us together to achieve it. In volleyball, my role was the setter, which to many is considered a lackluster position as we rarely attack the ball and score points with power. However, as a setter, my role is to set the pace, strategize and dictate the game from my team’s perspective. There is a long sequence of crucial events before a “kill” in volleyball and I strategized my teammate's individual strengths in both offense and defense to win. Anesthesia gives me the same opportunities to strategize anesthetics, balance individual patient’s comorbidities and anatomy all while maintaining a calm demeanor and level head through unexpected circumstances. In volleyball, I never shied away from tense games or difficult situations, instead I trusted in my own abilities and training despite uncharted territory. Lastly, I didn't need to actually score the point in order to understand my role and contributions to my team.

As an athlete, I understand the importance of practice and repetition which allow us to fail, but most importantly, to learn. I believe that the curriculum at this program will provide me with a well-respected education, which strongly reflects my learning style. I also admire the mandatory communication block in the curriculum because I believe an emphasis on clear and concise communication, is essential as an anesthetist.

Throughout the course of the next 5-10 years, I anticipate that both my husband and I will complete the next chapter in our educational pursuits. We both agree that [program name here] has the potential to nurture the next chapter in both our private and professional lives if given the opportunity.

What Makes This Sample Effective?

1. the theme is personal and consistent.

In this anesthesiology residency personal statement example , the author of this passage carries the theme of athletics throughout the statement. Having a theme can unify your personal statement and give it direction. This is a good example of a way to use a theme to tie together different ideas. Having a good theme is also something you should keep in mind when you’re answering anesthesiology residency interview questions , as program directors want to see that this particular specialty choice wasn’t simply drawn out of a hat; rather, your emphasis on a theme can demonstrate that your choice was intentional and the right fit.

2. The tone is positive throughout

Also, take note of how the author explained the transition to different schools without speaking negatively of the institutions. In your own personal statement, feel free to use the names of the universities you attended. They have been redacted here for anonymity. This statement has parts where you could customize it. Use the name of the program when possible or the name of the town. Taking time to add this into your statement shows the program that you pay attention to detail while personalizing it to each program.

3. Lessons learned apply to medicine

The writer of this personal statement relies on analogy to connect their experience to their interest in anesthesiology: “I understand the importance of practice and repetition which allow us to fail, but most importantly, to learn.” This analogy works so well because it shows why the applicant is suited to the program and specialty, it reveals an important aspect of their personality with evidence, and it sets expectations for how they want to contribute to the field. In your essay, you can use a similar strategy by tying together a major life theme or event with what you learned and how that applies to your medical training.

I was six years old when my father read to me the first chapter of “How Things Work.” The first chapter covered doors and specifically, the mechanics in a doorknob. What lay hidden and confined in the door panel was this complex system that produced a simple action. I credit this experience as the onset of my scientific curiosity and eventually my passion for complex systems found in medicine. Intensivists vigilantly maintain homeostasis within the human body, a complex system in and of itself, a concept I recognize as personally fascinating and enticing. I find myself especially drawn to the field of critical care and intensive care medicine. My dreams to become an intensivist would be highly complimented by a residency in surgery.

In critical care, each patient in the ICU is usually in a general state of shock. From the initial state of shock, the patient can be further complicated with comorbidities and chronic diseases that may require further intensive medical intervention so that they may recover from a recent surgery or traumatic event. This dynamic nature of the ICU is not available in every unit of the hospital and the high level of acuity does not suit everyone. I, however, enjoy the high energy of the enthralling, engaging and exciting environment offered by the ICU. I am personally energized and awakened by managing patients with surgically-altered physiology coupled with comorbidities. There is an overwhelming satisfaction when a patient following a bilateral lung transplant gets up from his bed and walks through the unit after days of being bedridden, or the moment we can discontinue the lines we had the patient on and finally talk to them after two weeks of intubation and sedation. Being in the ICU also encompasses the emotional seesaw of going from a successful patient case to a room in which a family has just decided that comfort care is the best way to proceed, which gives me chills just to type and verbalize.

The work of an intensivist is not only limited to the patient, but also the emotional well-being of the patient’s family as well. My involvement in the ICU has taught me that sometimes it is necessary to talk to a patient’s family, to explain to them simply that the postoperative expectations that they had had, may not be met. Communication is key in this field, both with the patients and the physicians of the OR. Communication prevents perioperative complications, establishes a willingness to follow directions and relays professionalism. It is important for an intensivist to have an excellent understanding of surgical procedures, so that they may explain to the patient what to expect as well as ease the nerves of the patient preoperatively. A surgical residency would facilitate this understanding and undoubtedly prove to be useful in my future training.

Studying medicine in Europe has taught me volumes about myself, how driven, motivated and open-minded I can be. To move so far away from home and yet be so familiar with the language, I feel blessed to be able to say that I’ve had a high level of exposure to diversity in my life. The mentality in [insert country name here] is if you don’t see the doctor, you are not sick. This common thought has to lead to an outstanding environment to study medicine and to see end-stage, textbook presentations of various pathologies and their management. Studying medicine in two languages has in itself taught me that medicine is a language and that the way a patient presents, conveys themselves, and the findings of the physical examination, all represent the syntax of the diagnosis. This awareness has reminded me that patient care, relief of patient suffering and illness, transcends the grammatical rules of the patient’s native tongue. My clinical experience in [insert country here] will aid me in providing thoughtful care to my future patients.

All things considered, I am ready to leave my home of the last four years and come back to the United States, to enter the next stage of my life and career. I am ready to work harder than ever, to prove myself to my future residency program and most importantly, learn so that I may be a suitable candidate for a future fellowship program in critical care. My experiences abroad have constantly pushed me to new horizons and encouraged responsibilities that I don’t believe I would otherwise have. I’ve developed a new level of human connection through my work in the ICU, the OR and my travels throughout Europe. These experiences will aid me in working with a diverse patient population and a diverse team of physicians. I hope [the program name here] can give me the variety and the background in surgery that I will need to succeed.

1. Atypical experiences are justified

This surgery personal statement example has to do double duty for the admissions committee. It has to explain why surgery, what this student can offer, and why this student is passionate about the field while simultaneously explaining why the applicant chose medical school abroad. If you are applying to a country where you did not attend medical school there, you have to explain why you studied abroad. This often poses a challenge for students. Be honest and positive about your experience. This student did an excellent job of explaining why it was such a good fit for their personality while highlighting the advantages of this experience.

Focus on the characteristics you gained from your experience abroad. Explain how your experience will translate into success in your residency. There are many things to be gained from having spent time outside of your home country. Talk about the skills you developed from living abroad. Unique details like those will set you apart when you are writing your statement.

2. Makes unique experiences an advantage

This applicant studied abroad in Europe. The way they talk about it is key: they explain how the experience was a challenge that they learned from. Most programs and schools are looking for medical school graduates who can contribute to their vision of diversity. If you have experience travelling abroad, this is a good chance for you to explain how this enriched your perspective and professional capabilities. Some of the skills that this applicant discusses are assets for a career in medicine: speaking two languages, exposure to diverse people and methods, and the ability to work with a large patient and physician population from different backgrounds. If you endeavor to explain some of your diverse experiences, be sure to make it clear what you gained and how you can apply it to your residency training.

3. The writer’s voice and style are unique

To get matched to the program and school of your choice, you will need to stand out from the crowd. To do this effectively in your personal statement, give your writing a unique style and allow your personality to shine through. In this example, the writer achieves this in the first paragraph in the “hook” in which they describe when their father used to read “How Things Work”; this life event left a lasting impression, and the writer links this to why a residency in surgery would benefit their goal of becoming an intensivist. With a first draft, it’s okay to experiment with word choice and content. Make sure you include all the necessary elements and formatting requirements, but try your best to put the “personal” in personal statement. Note that this is a general surgery example; if you were applying for plastic surgery or neurosurgery, you should read plastic surgery residency personal statement examples or neurosurgery personal statement examples for a slightly varied essay strategy.

Writing a residency personal statement? Here are the top books for residency applicants:

Residency Personal Statement Examples #4: Emergency Medicine

One of the most surprising things that I learned through my emergency medicine (EM) electives is that working in an emergency department is like leading a horse. I grew up on a farm in the [name of city], and working with animals was very much a part of my childhood. When walking a horse, one must be prepared for anything should the animal become spooked. It can startle at any moment and one must react quickly and calmly to redirect the thousand-pound creature. Similarly, in EM, one never knows when the department is going to become “spooked” by what comes through the door. EM is exciting, with a variety of patient presentations and medical procedures done on a daily basis. I enjoy dealing with the unexpected challenges that arise in caring for patients with backgrounds vastly different from my own. It would be a privilege to gain the skills as an emergency physician to provide acute life-saving care, to connect patients with resources and other healthcare professionals, and to provide comfort to patients and families in the settings of acute loss or difficult diagnoses. I feel that the [name of program] is the ideal path to reach that goal.

First, the [name of program] offers additional support and training to continue to perform research and other scholarly activities. Through my experience in quality improvement, I have learned of the value of research and how it can be applied to practical problems. For instance, while volunteering in a pool rehabilitation program for individuals with neurological disabilities, a patient who I had worked with for a year tragically suffered a fall and broke his hip leaving him significantly disabled. This led me to research inpatient falls during medical school and I initiated a quality improvement project and presented at several conferences, quality improvement rounds, and meetings with hospital stakeholders. After several years of work, I am very proud that this led to the implementation of a province-wide quality improvement initiative funded by [name of organization]. This initiative is physician-led and is aimed at reducing inpatient falls across [name of city]. This project demonstrated how rewarding research is when it can be translated into tangible initiatives and is why I am particularly interested in quality improvement research. I look forward to more dedicated time in the [name of program] to develop my research skills and to apply quality improvement to EM.

In addition to increased training in research, the [name of program] offers the opportunity to subspecialize within EM. While in medical school, I helped my single mother raise my much younger siblings and this has inspired my interest in pediatric EM. I maximized my studying through the effective use of weekly group study sessions and podcasts to allow for free weekends to return home to spend with my brother and sister. Through my experiences teaching and playing with my siblings, I have learned to deal with children in a calm and friendly manner. I used these skills to maintain positive therapeutic relationships with children during my pediatric EM rotation at [name of hospital]. For instance, I was able to cast the forearm of a frightened child by first demonstrating the procedure on her toy rabbit, and then calmly fitting a cast on her arm. I enjoy the emphasis on patient and family education as well as the focus on making the patient feel safe and cared for. I would love to explore this field further as my niche within the [name of program] in emergency medicine.

Alongside research and pediatric EM, I am also interested in teaching. Some of my fondest memories involve the evening teaching sessions during primary and secondary school spent with my grandpa, a retired teacher. My grandpa modeled effective teaching techniques, first assessing my knowledge and then expanding on it by asking questions and providing guidance when needed. Similarly, some of my best memories in medical school include the five-minute bedside teaching sessions after interesting cases that were taught in that way. Inspired by many residents and staff I have worked with, I look forward to expanding my teaching role in residency. Like my grandpa and my clinical mentors, I hope to help future students maximize their learning potential through the delivery of lectures and bedside teaching. Training within the [name of program] would allow additional time to develop the skills necessary for this, through increased exposure to mentorship, teaching role models, and opportunities to be involved in curricular development.

I would feel privileged to join the resident team in the [name of program]. I was fortunate that most of my core clerkship training including EM, as well as my fourth year EM elective, was at the [name of hospital]. What stands out the most to me most about working in the [name of hospital] is the tight-knit community feel in the setting of a high volume, high acuity ED. I value that the small program leads to a cohesive resident group and staff who are invested in their learners. Furthermore, from my rotations there, I know the ample procedural and hands-on exposure residents get from the beginning of their training. With my interest in pediatric EM, I value the longitudinal exposure to pediatrics at [name of program], with opportunities to do dedicated pediatric rotations both at [name of hospital], as well as [name of hospital]l. Finally, the [name of city] is my home; my family and friends are here, and I love the hiking, fishing, kayaking, and snowboarding that are all less than an hour away. I would be incredibly honored to have the privilege of pursuing EM in the [name of program], and look forward to serving my community.

Read some more Emergency Medicine Personal Statement Examples !

---------------------

The thought of caring for severely ill children seemed disheartening and overwhelming when I first began shadowing [name of doctor] at [name of hospital] five years ago. I was very nervous. While some of the cases were indeed difficult, my experience was starkly different. In one of our first cases, I quickly jumped in to comfort a scared child suffering from kidney disease. The mother of our patient confided in me about her son's struggles with bullying due to the disfiguring edema. I felt how much she appreciated being able to share her son’s challenges with me. Throughout my clinical experiences, I saw that caring for a pediatric patient often involves delicately navigating complex social situations and family dynamics. From that point on, I knew I had both the passion and compassion to succeed as a future pediatrician.

I am particularly keen to complete my residency at the [name of school], because I had such an immersive learning experience completing 5 years of research with [name of doctor] at [name of hospital] and at [name of hospital], not to mention [name of school]'s stellar international reputation. The incredibly high standard of excellence at [name of school], as well as [name of city] being my hometown, make the [name of school] my top choice to complete my residency. To further demonstrate the excellent education, I remember a time while shadowing at [name of hospital] in the genetics clinics where we discussed the pathophysiology of Bartter’s syndrome. The residents were having a hard time understanding this disease, but [name of doctor] explained the exact pathophysiology and downstream effects of it. The incredible intellect, mentorship and leadership [name of doctor] demonstrated has inspired me to pursue a nephrology fellowship upon completion of my residency.

During my elective rotations in [name of cities], I saw indigenous pediatric patients with a variety of ailments from hypoglycemia to cystic fibrosis. I spoke with them about the struggles of travelling long distances to obtain care. As an Inuit member of the [name of group], I have spent time reflecting on the medical needs of this much-overlooked population and hope to explore ways of reaching out to underserved populations in my future career.

I am prepared to be a leader and engaged learner in my residency program because of my participation in impactful leadership roles. I am currently the president of the [name of society], where one of my main duties is coordinating the [name of initiative], an initiative that teaches children about hospitals and healthy living. I was able to spend one-on-one time with disabled children teaching them about the heart through dance and instruments and activities to decrease fears associated with hospitals. This demonstrated the importance of promoting health care initiatives for kids and educate families and their children on how to be advocates of their own health.

As a competitive Irish dancer for sixteen years, I developed perseverance, determination, and time management that have been critical throughout my medical school training. Competing in front of judges and thousands of spectators all over the world, performing to my best ability under intense pressure was a necessity. I persevered with the challenge of competing at an international level and still maintained a very high level of academic performance while achieving my career high of second at the World Championships.

As an IMG applicant born and raised in [name of city] and educated in [name of country], I believe that my international education provides many advantages. I was exposed to diverse cultures and innovative ways of thinking from teachers from all over the globe at the [name of college] that I hope to bring back to Canada with me. Through the last 6 years, I have also had many research experiences and clinical electives in Canada that have given me insights into the intricacies of the Canadian Health Care system.

I am confident that pediatrics is the field I wish to pursue and I cannot wait to begin my residency so that I can start becoming an excellent clinician who advocates for children, as well as a scholar involved in research projects that will help advance the field. After successfully completing my pediatric residency program, I plan to pursue a pediatric fellowship. I am excited at the prospect of working and learning at the [name of school] while being an active and professional member of your residency program. I am also looking forward to developing my teaching skills and contributing to the community while also enjoying bike rides down the paths in the [name of path] and to be reunited with my [name of city] based family.

Want to see more Pediatrics Personal Statement Examples ?

“Code blue, electrophysiology laboratory” a voice announces overhead during my cardiology rotation. As the code team, we rush to the patient, an elderly man in shock. Seamlessly, we each assume our preassigned roles. I quickly review his chart and note to the team-leader that this patient had a previous EF of 10 percent and just got cardioverted. Vasopressors administered, intubation, central line secured, and the patient is stabilized and sent to our floor. During my rotations in internal medicine, I was constantly elated by my team’s ability to come together at such key moments. This gave me a sense of joy I did not find in other rotations. Moreover, I had inspiring attending physicians and residents who served as my mentors. They taught me that an internist is a medical expert committed to evidence-based medicine and perpetual learning, a compassionate physician, and an engaged community member. These lessons and the satisfaction of managing highly complex cases with a dedicated team consolidated my interest in internal medicine.

Compassion and a holistic approach to medicine remain quintessential for patient care. During my rotations, I took advantage of opportunities to learn from my patients both at the bedside and through independent reading. As a senior student, I prepared learning capsules that I presented to my team. This taught me to synthesize and communicate information efficiently. Beyond that, I took courses outside of the formal curriculum such as a point-of-care ultrasound course to improve my ultrasound procedural skills. When we no longer had any curative interventions to offer patients, I learned that acknowledging the patients’ suffering and being present for them in their most vulnerable time can ease their pain. As a resident at [name of school], I will continue my dedication to academic excellence and compassionate, patient-centered care in my efforts to care for my patients.

I have built strong ties to my community serving as president of the [name of school] Biology Student Union. Together, we enacted a complex study space and locker initiative through my role as a mentor at [name of organization]. These experiences instilled in me the values of proactivity and advocacy which I aim to bring with me to [name of school]. There, I hope to continue my community engagement as a mentor with the Big Brothers Big Sisters of [name of city]. Moreover, as I learn more about [name of town]'s healthcare system, I hope to combine that knowledge with my medical education to add my perspective to health policy decision-making in the province.

In addition to its excellent academic reputation, [name of school]’s commitment to academic excellence and continuing education, as exemplified by the abundant academic teaching, drew me to the program. Moreover, given my belief that we develop to be an amalgam of characteristics and values our mentors espouse, I was delighted to learn about the mentorship opportunities available. This was a unique characteristic that motivated me to apply to [name of school]. Finally, having lived in [name of city] for the last ten years, I am looking forward to spending the next chapter of my life in a smaller, more tightly knit community of [name of city].

As I learned and modeled the different roles of an internist, I also learned a lot about myself. I learned of my thirst for knowledge, of my desire to treat as well as to heal the patient, and of my urge to be a leader in my community. These characteristics will play a defining role in my residency. I also learned of my passion for acute medicine. After my residency, I hope to further subspecialize in cardiology. As a future cardiologist, I aim to provide patient-centered care, conduct research, continue my community engagement, and act as a role model to future generation.

Get inspired with these Cardiology Personal Statement Examples !

Watch this to learn what red flags to avoid in your residency personal statement!

Residency Personal Statement Examples #7: Psychiatry

I grew up in a tight knit military family in a community struck with the stigma of mental illness. Throughout my childhood we lost friends to the complications of untreated mental illness including overdose and suicide. I knew at that point that I wanted to pursue mental illness and completed a psychology degree and then a nursing degree. In University, I volunteered in a distress service for 6 years, providing individual sessions to students on issues including suicidality, interpersonal violence and addiction. As a registered nurse, I honed my skills in mental status examinations and cared for their comorbid psychiatric illness with medical disease utilizing communication and building rapport. I saw the impact of life altering conditions and procedures on their mental health. As a medical student, I continued to explore psychiatry through City X summer studentship and appreciated the breadth of psychiatric practice. As a clerk, I completed a range of psychiatric electives, caring for patients in multiple care settings and across various socioeconomic and age ranges. I enrolled in the integrated community clerkship, in X town, a community 900 km North of X city. The socioeconomic disparities and lack of access to mental health services had a negative impact on community, with suicidality and addictions. I followed my patients across practice domains assessing their functioning, medication regimen and continued to build a collaborative relationship. This proved crucial to uncover their health status across domains and helped me identify areas to support their challenges. 

I value the ability to understand my patients from a biopsychosocial framework and addressing negative thought processes in support of their wellness. I view our duty in psychiatry is to support their strengths on a trajectory to wellness and provide guidance and resources utilizing pharmacological and non-pharmacological therapies. Psychiatry is a newer field of medicine, allowing for ongoing innovations in treatment and practice. This is exciting to explore novel approaches to treatments as we continue to uncover the physiological, neurological and pharmacological dimensions of mental health. It is also important to recognize the challenges of psychiatry. The history of mental illness creates access to care barriers from both a structural viewpoint with longer wait times and on a personal level due to their concern about the social and occupational implications of stigma. As our population ages, this threatens to overwhelm the current psychiatric infrastructure and will require more complex approaches due to medical comorbidities and medication contraindications. We will require ongoing research focused on medical comorbidities of neuropsychiatric illness and treatment modalities to improve quality of care. 

I am drawn to the University of X psychiatry program due to its resident focused approach. I appreciate the ongoing mentorship and supervision and the preparatory endeavors including the mock examinations. From a clinical perspective, the program has a strong psychotherapy curriculum and offers unique elective opportunities including electroconvulsive therapy. The ability to continue serving rural communities solidifies my interests in this well-known program. 

Check out these Psychiatry Personal Statement Examples !

Want help with your residency personal statement? Hear what our students have to say:

Residency Personal Statement Examples #8: Internal Medicine

“People are drawn to medicine in one of two ways: the humanity or the science.” My mentor, [name of doctor], staff medical oncologist at the [name of hospital], once told me this. As a volunteer during my premedical studies, I assisted him with his impromptu lunchtime clinics while others were on break and was able to catch a glimpse of his patients’ unshakable trust in him. Those moments sparked my interest in Internal Medicine. Internists are entrusted with the most complex patients in any hospital. Therefore, Internists take on the responsibility of a patient’s trust in their lowest, most disoriented moments. Accordingly, when I finally started clinical rotations, I saw it as my responsibility to fully understand each patient’s motivations and fears to advocate for their goals. One patient I had gotten to know still stands out in my mind. She was 95, witty, and self-assured but was found to have bone metastasis causing excruciating pain during her hospital stay. She knew she did not want aggressive life-prolonging treatment and declined further workup, but how could we help her? I suggested palliative radiotherapy to my team because I remember her telling me “I had a good life. I am not scared of death, but if I have to be around for a while, can’t I be more comfortable?” Therefore, my team entrusted me to talk to her and her family about a referral to Radiation Oncology. She responded to me with “I don’t think there’s anyone who knows what I’d want better than you. You’ve listened to me so much. I trust you.” I spent the next half hour explaining the rationale behind the referral to both her and her family. She received urgent Radiotherapy two weeks later. Her narcotic requirement decreased by more than half. After that moment, I envisioned that one day, I could also look into the eyes of someone at their most vulnerable moment and give them confidence to trust me and my team with their care.

Although my interest in Internal Medicine is rooted in the human connection, my attention to detail, work ethic, and natural curiosity, also makes me especially well-suited for the challenges of Internal Medicine. Indeed, beyond the human connection, Internal Medicine’s challenges of complex problem solving, and large ever-growing breadth of knowledge is also what makes each day so satisfying. When I was on the Nephrology Consult service, I was following a patient with a kidney transplant who was admitted for Line Sepsis. I noticed a mild Non-Anion Gap Metabolic Acidosis and a persistent mild Hyperkalemia. I presented my findings to my staff as a possible Type 4 RTA. He complimented me on my attention to detail and warned that a Type 4 RTA in a kidney transplant patient could be a sign of rejection. We restarted his anti-rejection medication that had been held due to his infection, his electrolyte abnormalities corrected in less than two days. My attention to detail is a particular asset for Internal Medicine because more than any other specialty, the tiniest details like a mildly abnormal lab work, when pieced together in the correct way, could solve the most difficult clinical problem. That is also what makes problem-solving in Internal Medicine so satisfying. My mentors have always complimented me on my work ethic. However, I enjoy staying late for admissions and additional learning or reading hours around my patients at home because learning Internal Medicine is so interesting.

On the other hand, Internists are also tasked with the very large, working with multiple professionals and navigate system issues to keep patients healthy and out of hospital such as when [name of doctor] entrusted me with planning the discharge of a homeless patient during my Medicine CTU elective at [name of hospital]. The patient had Schizophrenia and Grave’s Disease and had been admitted to hospital multiple times that year with thyrotoxicosis due to medication non-adherence. During his admission, I had elicited the help of two homeless outreach coordinators to ensure proper follow-up. Therefore, by the time of discharge, he had a new family doctor, timely appointments with the family doctor and endocrinologist, maps with directions to each appointment, his prescription medications ready to go, as well as a new apartment application.

Ultimately, I am fortunate to be drawn to Internal Medicine for both its humanity and science. I believe that I have the qualities that will help me excel in its smallest details and its largest responsibilities. In residency, I aim to explore and learn as much Internal Medicine as possible before becoming an expert in one area so I can make an informed choice and be a well-rounded physician. Therefore, the fact that [name of city] has so many leading experts especially suits my learning goals. Indeed, during my electives in [name of city], I’ve already learned knowledge that I’ve not encountered elsewhere like the Bernese method of Buprenorphine induction. The availability of resources such as the DKA management simulation and the use of presentations of cutting-edge knowledge as part of evaluation also suits my self-directed learning style. Furthermore, my research has focused on the PMCC Gastro-Esophageal Cancer Database where we were able to discover various new details in the clinical behavior of Gastro-Esophageal cancer due to the large volume of patients are PMCC and its world-class expertise. This line of research would not work as well anywhere else in [name of country]. Indeed, our database is currently the second-largest in the world. Therefore, the second reason [name of city] is my ideal place for training is for its unique research opportunities, so I can continue to contribute to further medical knowledge. Lastly, [name of city] is the most diverse city in [name of country]. Growing up as an immigrant, I had experienced how cultural backgrounds can become a barrier to receiving good medical care. Therefore, the diverse patient population and strong allied health support in [name of city] could also allow me to hone the skills required to assist me in providing good quality care to all patients, regardless of background.

Here are more Internal Medicine Personal Statement Examples !

My first exposure to Family Medicine occurred during my time as a Medical Officer working in a small clinic in Nigeria in fulfilment of the [name of service]. There, I recognized that a career in this specialty would offer me the opportunity to not only experience the aspects I cherished most about other specialties, but fulfill my personal interests in advancing community health.

My many encounters with patients during my days in the clinic reaffirmed my view of Primary care physicians as being on the frontline of diagnosis and preventive medicine. There was the middle-aged diabetic patient who had first presented to the emergency with diabetic ketoacidosis, the hypertensive man whose initial complaint of a persistent headache prompted the discovery of his soaring blood pressure, and the adolescent with a family history of allergies who was diagnosed with asthma. These encounters highlighted that as the first point of contact, the general practitioner is not only responsible for diagnosis, but often in ensuring patients are set on the path of healthy habits to prevent disease complications. This unique opportunity to significantly advance the well-being of a patient, and by extension, the community renewed my interest in the field.

An especially appealing feature of Family Medicine is that it provides an opportunity for patient care without limitations of age, sex, disease or organ system. From treating colds and routine checkups to referral for a suspected malignancy, I enjoyed that every day in the clinic was a learning experience and no day was routine. In addition, having a diverse population of patients and cases requires an abundance of clinical knowledge and I cherish the chance to learn and expand my skills every day.

I also value that an essential part of Primary care is in the enduring relationships the practitioners develop with patients. I recall several moments during my clinical experiences when I recognized that some of the bonds formed during ongoing patient interactions had evolved into lasting friendships. Being a practice of continual care, I appreciate that this specialty provides many opportunities to follow patients through different stages of their lives ensuring a deepening of relationship and compliance with care. I was inspired during my clinical rotation here in the United States when I saw how my preceptorís long-term relationships with patients enabled their compliance and often extended to different generations within one family.

Ultimately, I am confident that my experiences have prepared me for a career in this specialty. An agreeable, attentive and compassionate nature has aided me in gaining trust as well as building meaningful interpersonal relationships which are crucial components of this field. Furthermore, my interaction with an extensive array of patients during my clinical and volunteer experiences has equipped me with the ability to communicate and relate to patients across different age groups and backgrounds. In addition, I enjoy working to coordinate patient care with colleagues and other specialties and value that the wellness of the patient is a result of hard work, dedication, and teamwork.

Thus, I hope to find a residency program dedicated to providing in-depth clinical training with a diverse patient population and an emphasis on health promotion and disease prevention through patient education and community service. Moreover, I look forward to being part of a program that will encourage my pursuit of intellectual development and advancement to enable my transition into a well-rounded, competent and skilled physician committed to serving people with needs in all areas of medicine. With a career in this specialty, I know that every day will bring a new opportunity to influence health behaviors, and while there will be challenges, fulfilling them will always be satisfying.

Here I am, yet again. Last year, I also applied for a position as a dermatology resident. Though I was not selected, I return with the same diligence and perseverance, as well as additional skills and knowledge. My continued dedication to pursue a career in dermatology reminds me that no good thing comes easily and pushes me to stay motivated and work hard toward my goals. 

I am drawn to dermatology for a host of reasons, one of which is the opportunity to work with my hands. In my current residency program, I have had the opportunity to assist in various surgical procedures. I recall the subdued exhilaration I felt when removing my first lipoma and the satisfaction of observing the surgeon completed the procedure with precision and care. My excitement for surgery continued to be reinforced in the many subsequent procedures I assisted with and I look forward to honing my surgical skills further as I complete my training in dermatology. 

However, to me, “hands-on” is defined as more than just its literal meaning. The opportunity to build relationships with patients steers me more towards a career in outpatient medicine. During my dermatology outpatient rotation, I was involved in the care of a patient who presented initially complaining of a heliotrope rash and gottron’s papules. When she expressed a deep sense of shame about this rash, I became acutely aware of how patient’s external disease can influence their internal emotions. I thus responded empathetically, simultaneously validating her concerns and providing her with much-needed assurance. When she was later diagnosed with dermatomyositis secondary to underlying breast cancer, this patient requested to speak to me specifically, recalling the positive interaction we had shared before. Again, I was able to explain the diagnosis and treatment plan with patience and regard for her every concern. Developing a trusted physician-patient relationship is crucial in the field of dermatology because most patients exhibit strong internal emotions from their visually external disease. Also important is the ability to deliver difficult news and be considerate of patients’ feelings in these delicate moments. I plan to continue to use these skills during my career as a dermatologist.  

To me, dermatology is also a field that is thought-provoking and stimulating due to its constant evolution and advancements. Thus, during my internship, I committed to educating myself in the field of dermatology through multiple research projects. My research thus far has been focused on whether UV light lamps used in gel manicures increases the risk of skin cancers as well as the outcomes of using intralesional 5-fluorouracil for squamous cell carcinoma and keratoacanthomas. While my research was focused in the field of dermatology, I did not hesitate to take on additional projects, pursuing assignments in both breast cancer and hemophagocytic lymphohistiocytosis. I strongly believe the best doctors have a thorough understanding of the practice of medicine in totality as our ability to incorporate this knowledge in our diagnosis and treatment of our patients directly impacts their wellbeing. For these reasons, I strive to continually educate myself in not only dermatology, but other fields that might have implications on my practice. 

My ideal dermatology program would allow me to manage a variety of complex medical dermatological conditions and engage in research, both of which will continue to challenge me intellectually and push me to exercise creativity to develop innovative solutions to dermatological treatments. As someone who enjoys working with my hands and the instant gratification of the surgical approach as a treatment option, I would also value the opportunity to perform surgeries and improve my surgical skills. Furthermore, I have found that beyond medicine, the people in each program make or break an experience. Positive attitudes, expressed dedication, and mentorship are vital characteristics in any program of my interest.

I am confident my aspirations will be fulfilled in the field of dermatology, but more importantly, I know I will be a good contribution to this field and your program – my work ethic, motivation, and commitment unwavering. I am determined, impassioned, and excited to embark on this next phase of my journey. 

Check out even more Dermatology Personal Statement Examples !

10 more residency personal statement examples, residency personal statement example #11, residency personal statement example #12, residency personal statement example #13, residency personal statement example #14, residency personal statement example #15, residency personal statement example #16, residency personal statement example #17, residency personal statement example #18, residency personal statement example #19, residency personal statement example #20.

Want some more residency personal statement tips?

How To Address Areas of Concern

There are some things that are out of our control. Sometimes we have to take time off to deal with personal issues, or sometimes we have to retake tests. If you have something you feel like you need to explain in your application, the personal statement is the area to address it. If you had a leave of absence or failed an exam, you should offer a clear, unemotional explanation of the situation. Use positive language. Whatever the area of concern, try and phrase it in the most favorable light. Take accountable for what has happened, but do not place blame or make an excuse. Here are some phrases you can try and use in your personal statement.

Sometimes we have to interact with people who we don't see eye to eye with. When I worked with (you can choose to say the person's name or just use their title) I learned how to (insert a lesson here). Even though it was a challenge, I have gained skills that will better my future practice. ","label":"Unfavorable Evaluation by an Attending","title":"Unfavorable Evaluation by an Attending"}]" code="tab1" template="BlogArticle">

Keep in mind that these are suggestions. If you are concerned about an area of your application that might be a red flag, it may be in your best interest to address it head-on. The choice to write about them is your own individual opinion. Your personal statement should highlight the best side of you. If you think that an area of weakness might hurt your chances, it may be beneficial to take ownership of the problem and write it in a way that will show what you learned and how it made you better.

For the most part, your residency personal statement should be within a one-page limit or approximately 750-850 words. Be sure to check your specific program requirements to verify before you begin writing.

It's entirely up to you if you want to address unfavorable grades or gaps in your studies. However, if you feel something in your application will be seen as a red flag, it's best to address issues head-on instead of having admissions committees dwell on possible areas of concern.

If you're going to address a gap, just ensure that you have a clear narrative for why you took these breaks, what you did on break and what this break means for your ability to function at a very high academic level for many years to come.

If you're addressing a poor evaluation, ensure that you take responsibility for your grade, discuss what you learned and how your performance will be improved in the future - then move on. It's important that you don't play the victim and you must always reflect on what lessons you've learned moving forward.

Absolutely. While it's not necessary to discuss your personal connection to a program location, showing program directors that you have ties to their program's location can give you a competitive edge over other applicants. The reason being is that it's a way to show program directors that you are invested in practicing medicine locally.

That's not to say that you have to apply to programs that are within your home state or province, but if one of the reasons you love a particular program is because of its location in your hometown, don't be afraid to mention this. Whether you enjoy the outdoor activities in the program's location, have family and friends in the area, or even grew up in the area at some point, these can all be great aspects to mention.

Firstly, it's important to check the program's specific requirements for your statement because some programs have a specific prompt or multiple prompts that you'll need to address. If you are not given a prompt, in general, your statement needs to answer “why this specialty?” and “why this program?”. Your responses must be supported with your personal experiences and your statement should incorporate your future career goals.

No, instead you'll be preparing one personal statement for each specialty. For example, if you're applying to emergency medicine and family medicine, you'll need to prepare one statement for emergency medicine and one statement for family medicine.

As long as it's during the application season, you can edit and review your personal statement. However, keep in mind that if you edit your personal statement, there is no guarantee that programs will review the most up to date version. For this reason, it's best to only assign your personal statement to programs once you've 100% happy with the final version.

No, there is no limit on how many personal statements you can create. 

Your personal statement should have three major structural elements: the introduction, the body, and the conclusion. Your thesis statement will appear in your introduction in the first paragraph. The body is for you to discuss major experiences relevant to your chosen specialty, and the conclusion is generally the place to summarize and highlight some of the item you mentioned in the body or introduction.

Want more free tips? Subscribe to our channels for more free and useful content!

Apple Podcasts

Like our blog? Write for us ! >>

Have a question ask our admissions experts below and we'll answer your questions.

it is comprehensive and precise

BeMo Academic Consulting

Hi Shiva! Thank you very much for your comment! Glad you found these helpful.

Get Started Now

Talk to one of our admissions experts

Our site uses cookies. By using our website, you agree with our cookie policy .

FREE Training Webinar: 

How to make your residency application stand out, (and avoid the top 5 reasons most applicants don't match their top choice program).

personalized personal statement eras reddit

Documents for ERAS® Residency Applicants

New section.

Each program sets individual requirements for the documents that should be submitted with the MyERAS® application. Be sure to research each program individually to determine those requirements before making document assignments.

  • Sign In to the MyERAS® Portal
  • ERAS® Timelines for Residency Applicants
  • ERAS Participating Specialties and Programs

Within your MyERAS account, you may create personal statement(s); identify the people who will write letters of recommendation (LoRs); authorize the release of your Comprehensive Osteopathic Medical Licensing Examination (COMLEX-USA) and/or United States Medical Licensing Examination (USMLE) transcripts; and manage all other documents. Below is an overview of each of the main documents used in ERAS. For detailed information, please see the Documents section of the MyERAS User Guide.

Tracking Documents

The dashboard provides a snapshot of the progress of various documents and their statuses in the MyERAS portal.

  • Uploaded but Unassigned LoRs  - Count of LoRs that have been uploaded but are not assigned to any programs, highlighting that assignments may need to be made.
  • Unassigned Personal Statements  - Count of Personal Statements that have been saved but are not assigned to any programs, highlighting that assignments may need to be made.
  • Latest USMLE Request Status  - Current status of the latest request made to NBME or ECFMG (for IMG Residency) after at least one program has been applied to with the USMLE transcriptassigned.
  • Latest COMLEX-USA Request Status  - Current status of the latest request made to NBOME after at least one program has been applied to with the COMLEX-USA transcript assigned.
  • Status of Additional Documents  - Status of all other applicable documents as either  Not Uploaded  or  Uploaded .

The  Photo  is most often used by programs to help identify applicants when reporting for an interview. Applicants must upload their own  Photo  in the MyERAS portal by selecting  Upload New Photo  in the  Actions  column. A photo file should not exceed these requirements:

  • Dimensions: 2.5 in. x 3.5 in.
  • Resolution: 150dpi
  • File Size: 150kb

Personal Statement

The personal statement may be used to personalize the application to a specific program or to different specialties. There is not a limit to how many personal statements you may create; however, you may only assign one (1) for each program.

Note : There are a number of websites that provide examples of Personal Statements. Do not copy any information from these sites and use them in your Personal Statements without giving credit to the author. This is considered plagiarism. See the  ERAS Investigation Policy

Special Note About Formatting

  • Personal Statements must be created in plain text formatting. HTML and other special text formatting, such as bold, italics, underline, text color, and alignment, are not allowed. Personal statements created outside of the MyERAS system should be done in a plain text word processing application such as Notepad (for Windows users) or Text Edit (for Mac Users) to ensure text stays as clean as possible.

LoRs must be uploaded through the ERAS Letter of Recommendation Portal (LoRP) by the  LoR Author . It is the applicant’s responsibility to follow up with  LoR Authors  regarding LoRs.

  • Creating LoR Entries  - You must create a LoR entry for each LoR you intend to use during the application season.
  • Confirming LoR Entries  - You must confirm a LoR entry before an associated  Letter ID  can be generated.
  • Uploading LoRs  - The Letter ID contained in the LoR Request form must be used to upload the associated LoR through the Letter of Recommendation Portal (LoRP).
  • Resending New Scores  - Applicants must take action in their MyERAS portal to resend USMLE scores to programs previously designated to receive them.

The USMLE transcript is required by many MD residency programs as part of an application to be considered for their positions.

  • Authorizing the Release for the USMLE Transcript  - Applicants must authorize the release of their USMLE transcript in order to make assignments of the USMLE transcript to the programs they designate.
  • Paying for the USMLE Transcript  - The NBME or ECFMG (for IMG Residency) charges a one time fee of $80 for transmitting USMLE transcripts to the programs designated by applicants.
  • Viewing the USMLE Requests Status Report  - Applicants can view the USMLE Requests Status Report to track the status of their USMLE requests by program.

The COMLEX-USA transcript is required by many AOA-accredited and ACGME-accredited residency programs as part of a D.O. applicant’s application to be considered for their positions.

  • Authorizing the Release for the  COMLEX-USA  Transcript  - Applicants must authorize the release of their COMLEX-USA transcript in order to make assignments of the   COMLEX-USA transcript to the programs they designate.
  • Paying for the  COMLEX-USA  Transcript  - The NBOME charges a one time fee of $80 for transmitting COMLEX-USA transcripts to the programs designated by applicants.
  • Viewing the  COMLEX-USA  Requests Status Report  - Applicants can view the COMLEX-USA Requests Status Report to track the status of their COMLEX-USA requests by program.

Uploading : An applicant’s Designated Dean's Office is responsible for uploading the Medical School Performance Evaluation (MSPE or “Dean’s Letter”) into the ERAS system for residency applicants. Find out more in the frequently asked questions section.

IMGs Only : IMG residency applicants must indicate in the MyERAS system if they themselves or their medical school will provide a MSPE to the ERAS Documents office at the ECFMG. Instructions for submission can be found here:  https://www.ecfmg.org/eras/applicants-documents-index.html .

Medical School (MS) Transcript

Uploading : An applicant’s Designated Dean's Office is responsible for uploading the  MS Transcript  into the ERAS system for residency applicants. Find out more in the frequently asked questions section.

IMGs Only :

  • IMG residency applicants must indicate in the MyERAS system if they themselves or their medical school will provide a MS Transcript to the ERAS Documents office at ECFMG. Instructions for submission can be found here:  https://www.ecfmg.org/eras/applicants-documents-index.html .
  • ECFMG Status Report:   The  ECFMG Status Report  confirms the ECFMG certification status for an IMG residency applicant. This report contains the month and year that examinations were passed for ECFMG Certification, but does not contain your USMLE transcript.
  • Uploading:  The ECFMG is responsible for uploading the ECFMG Status Report into the ERAS system for IMG residency applicants.
  • 2025 ERAS Webinars for Residency Applicants

Please contact the AAMC Support Center (ASC).

Send us a message.

Monday - Friday 8 a.m. - 6 p.m. ET 

Closed Wednesday, 3 - 5 p.m. ET

Please visit this page to stay up to date on ERAS news and announcements.

Use the free Residency Explorer™ tool to search and compare programs. Browse the AAMC's Careers in Medicine website to research specialties.

Google Chrome 

Mozilla Firefox 

Microsoft Edge 

Safari* 

*While most AAMC services  support certain browsers , the PDWS does not fully support Safari. We recommend Mac users use Firefox or Chrome.    

  • (888) 381-9509
  • [email protected]
  • Book a Meeting
  • student login
  • Student Login
  • Our Services
  • Our Story How it started
  • Our Team Meet Our Advisors & Tutors
  • Our Services How we can help you
  • Our Difference Learn why we stand out
  • Success Stories & Testimonials Hear the stories
  • For Parents Learn why you should trust us
  • In the News Read Our Stories
  • Frequently Asked Questions Find answers
  • MCAT Tutoring One-on-One Personalized Help
  • MCAT Go An Audio Learning Experience
  • MCAT Practice Exams Boost Your Score
  • MCAT Prep App Videos, Flashcards & Q-Bank
  • MCAT CARS Mastery Top-Rated CARS Video Course
  • Pre-Med Coach Early High School Roadmap Planning
  • College Admissions 11th & 12th Grade Pre-Med Consulting
  • Direct Med Advising BS/MD Application Support
  • Interview Preparation BS/MD Candidates
  • Pre-Med Coach Pre-Application Development
  • Application Advising Med School Admissions Support
  • Personal Statement Editing Refine Your Narrative
  • AMCAS Editing Application Editing
  • Secondary Editing Secondary Application Editing
  • Interview Preparation Realistic Practice
  • CASPer Preparation Simulation & Coaching
  • Ontario Application Support OMSAS Application
  • Residency Advising Complete Match Support

Residency Interview Preparation

  • ERAS Personal Statement Refine Your Story
  • USMLE STEP 1 Pass Your First Step
  • Shelf Exams Ace your rotations
  • USMLE STEP 2 Shine on your boards
  • USMLE STEP 3 Conquer your final hurdle
  • COMLEX LEVEL 1 and 2 Score higher
  • Institutional Partners Enhance your student offering
  • Organizational Partners Provide value to your students
  • Virtual Shadowing Explore Medical Specialties
  • Extracurricular Activities Apply now!
  • Under the Stethoscope Admissions Video Course
  • Research Roadmap Master Clinical Research
  • MSC Score Calculate Your Chances
  • Guidebooks Comprehensive Guides
  • Med School Explorer Find Your School
  • MCAT Masterclass Videos, Questions, Notes
  • Anatomy Anki Deck Practice with 4,000+ Flaschards

Residency ERAS Personal Statement Consulting & Editing

Our premium service is tailored exclusively for medical students on the journey to secure their dream residency. Our team of professional writers and experienced physicians collaborate to optimize your ERAS applications and finetune your personal statements, ensuring they stand out in the competitive pool. With a sharp focus on conveying your unique medical journey, skills, and aspirations, we translate your experiences into a compelling narrative, capturing the attention of residency directors. MedSchoolCoach is more than just an editing service; it’s your strategic partner, lending you an edge that propels you toward a successful career in medicine.

  • RESIDENCY & FELLOWSHIP PREP
  • THE PERSONAL STATEMENT & ESSAY EDITING PROCESS
  • meet your team

Customer Reviews </li-->

Talk to our Enrollment team

Craft the perfect eras personal statement.

doctor icon

RESIDENCY PHYSICIAN ADVISORS

message bubbles icon

WRITING ADVISORS

Man teaching about the meaning of graphs icon

INCREDIBLE SUCCESS RATE

note paper icon

PERSONAL ATTENTION

Residency & fellowship applications we can help with, specialties.

  • PGY-1 Transitional Year
  • PGY-1 Internship Year
  • All PGY-2 + Categorical Programs (such as dermatology, radiology, radiation oncology, PM&R)
  • All Categorical Programs (internal medicine, surgery, pediatrics, emergency medicine, all surgical subspecialties, etc)

Applications

  • Fellowship Applications

Invest in Your Medical School Future and Boost Your Odds of Getting Accepted!

Choose your advisor:

Icon

Steven Ruby MD Associate Director of Advising

Dr. Ruby is a distinguished vascular surgeon who has completed his general surgery residency at Columbia-Presbyterian Medical Center and a fellowship in Vascular Surgery at Harvard Medical School, and has held numerous leadership positions in various surgical societies. He is a founding member of Vascular Associates of Connecticut.    

Joel Ramirez MD, MedSchoolCoach

Joel Ramirez MD Associate Director of Advising

Dr. Ramirez is an integrated vascular surgery resident at UCSF who is passionate about teaching and tutoring for several board exams. He has served on the University of California, San Francisco School of Medicine admissions committee and is committed to being a leader in medical education.        

Susan Choo MD, MedschoolCoach Advisor

Susan Choo MD Associate Director of Advising

Dr. Choo completed her pre-med education at UC Berkeley and got her medical degree at UC San Diego. She is a board-certified pediatrician and has served on the admissions committee, reviewed applications, and worked with UC Irvine to teach and prepare students for the application process.    

Wendy Gammon, MedSchoolCoach Advisor

Wendy Gammon MA, MEd Director Writing Advisor

Wendy has worked as an admissions screener for the Internal Medicine Residency Program at Overlook Hospital, a pilot case developer with the NBME for the USMLE exam, and an assistant professor of medicine and director of the standardized patient program . She holds two master's degrees in English and education and has taught several AP courses.

Kachiu Lee MD, MedSchoolCoach Advisor

Kachiu Lee MD Director of Direct Med (BS/MD) Advising

Dr. Lee is a board-certified dermatologist and an assistant professor of dermatology at Brown University. Dr. Lee has a passion for medical education and also does research in developing new treatments for skin cancer.  She specializes in BS/MD admissions, with more than 95% of her clients getting interviews for BS/MD programs each cycle.  

James Weintrub MD, MedSchoolCoach Tutor

James Weintrub MD Associate Director of Advising

Dr. Weintrub trained in general surgery at Boston University, plastic surgery at McGill University, and microsurgery at the Texas Medical Center. As Chief of Plastic Surgery at the Providence VA Hospital & Clinical Asst. Professor of Surgery and Family Medicine at Brown, Dr. Weintrub has helped scores of aspiring physicians get accepted into medical school.

Ed Walsh MD, MedschoolCoach Advisor

Ed Walsh MD Associate Director of Advising

Dr. Edward Walsh is an Emergency Medicine physician who graduated from the University of Virginia School of Medicine. He is especially interested in medical education and preparing students for the challenges of medical school and beyond, also serves faculty member at the James Madison University Physician Assistant program.

Ziggy Yoediono MD, MedSchoolCoach Master Advisor

Ziggy Yoediono MD Associate Director of Advising

Dr. Yoediono received his MD from the University of Rochester, and did his training at the Harvard Longwood Psychiatry Residency Program. He has worked at Duke as a pre-major advisor and admissions interviewer. Dr. Yoediono co-authored papers published in The New England Journal of Medicine and Academic Medicine.    

Newsha Lajevardi MD, MedschoolCoach Advisor

Newsha Lajevardi MD Associate Director of Advising

Dr. Newsha Lajevardi is a board-certified dermatologist who practices medical, pediatric and cosmetic dermatology, cutaneous surgery, and laser surgery. She has a nontraditional path to medical school and is highly involved in the application and interview process for prospective dermatology residents.

James Kelly MD, MedschoolCoach Advisor

James Kelly MD Associate Director of Advising

Dr. Kelly attended Georgetown University for both undergraduate and medical school. He completed his Ophthalmology residency at North Shore/Long Island Jewish and is now in private practice, as well as helping Ophthalmologists prepare for their oral boards. He enjoys traveling, live music, and sports.    

Edward Lipsit MD, MedSchoolCoach

Ed Lipsit MD Associate Director of Advising

Dr. Lipsit is a Board Certified Radiologist with extensive experience in diagnostic ultrasound. Currently, he is an Associate Clinical Professor of Radiology at The George Washington University School of Medicine and Health Sciences and serves as an educational consultant. Dr. Lipsit has also been involved in admissions consulting for several years.    

David Flick MD, MedSchoolCoach

David Flick MD Associate Director of Advising

Dr. Flick graduated Magna Cum Laude from Loyola Marymount University and attended medical school at UC Irvine after receiving the Army health professions scholarship. He has served as a flight surgeon for the Army. While at the UC Irvine School of Medicine, he was an admissions committee member.

Renee Marinelli MD, MedSchoolCoach

Renee Marinelli MD Director of Advising

Dr. Marinelli has practiced family medicine, served on the University of California Admissions Committee, and has helped hundreds of students get into medical school. She spearheads a team of physician advisors who guide MedSchoolCoach students.

Need more comprehensive help for the match?

HOW OUR ERAS EDITING PROCESS WORKS

eye over globe icon

Your residency advisor will use our comprehensive intake form and spend time getting to know you on a personal level to find out what makes you unique.

cloud with arrow lines icon

Using your individual story, your Physician Advisor will work with you to brainstorm a cohesive narrative about the speciality of your choice.

mechanic tools icon

Develop and Organize

Once you have decided on content, your Writing Advisor will help you develop and enhance your story, turning your ideas into an organized and cohesive essay that puts your experiences in the spotlight.

funnel icon

Edit and Finalize

After a few drafts, your Writing Advisor will refine your prose and correct smaller writing errors that stand in the way of excellence. Your Physician Advisor will then approve the final product.

smartphone icon

With the help of your team of residency advisors, you will craft an ERAS application that will get you interview invites, and lead to the match outcome you want!

Meet a Residency Advisor

personalized personal statement eras reddit

Dr. Choo is a former Associate Program Director at the CHOC Children’s and UC Irvine residency programs

MedSchoolCoach has the experience:

12k+ students helped

Unmatched experience, meet some of our residency advisors.

Physicians with Residency Admissions Committee Experience

Edward Lipsit MD, MedSchoolCoach

Amar Mandalia MD Associate Director of Advising

Dr. Mandalia was a member of the University of Miami Admissions Committee, an accomplished medical writer and advisor to hundreds of successful applicants.

Henry Ng MD, MedschoolCoach Advisor

Henry Ng MD Advisor

A trailblazing physician and advocate, Dr. Ng has spearheaded LGBT health initiatives, founded Ohio's first LGBT-focused PRIDE Clinic, and achieved numerous accolades for his work in healthcare diversity and inclusion. Dr. Ng was an assistant dean and member of the Case Western Reserve University School of Medicine Admissions Committee.

Ed Walsh MD, MedschoolCoach Advisor

Megan Flynn MD Master Advisor

Dr. Flynn has been involved in both medical school and residency admissions committees and is dedicated to collaborating with anyone interested in pursuing medicine. She is passionate, compassionate, and excited to work with you.

Alexis Dallara-Marsh, MedSchoolCoach Advisor

Alexis Dallara-Marsh MD Advisor

Dr. Dallara-Marsh is an esteemed neurologist and epileptologist in Bergen County, New Jersey, with a notable academic journey, comprehensive training in child neurology and epilepsy, an active role in mentoring and academic admissions, and a research interest in Palliative Care. Dr. Dallara-Marsh has extensive admissions experience.

Want to Meet More of the Team?

Frequently Asked Questions

We have physician advisors across multiple specialities, and when appropriate, we'll do our best to pair you with one in the particular speciality you are looking at matching into. However, we cannot guarantee this, and there are times when another advisor may actually be a better fit!

With MedSchoolCoach, you get the advantage of having a Physician Advisor and Writing Advisor to help you develop your story. Our Physician Advisors have sat on admissions committees and have evaluated thousands of applications, so they understand exactly how to bring out the best in an applicant. Our Writing Advisors are professional writers and editors who will help you refine your concepts and create a compelling essay. This combination results in an extremely powerful team that will take your application essays to a new level.

Yes! All our essay editing packages come with brainstorming time with your Physician Advisor. Your Writing Advisor will also provide you with a worksheet to help you outline your narrative.

We use our advising portal, CHART, to organize your essays. When you upload your drafts to the platform, your Writing Advisor will review them in detail and then provide constructive feedback on how to improve them. As you get closer to a final draft, we will focus on things like word choice, sentence structure, and grammar.

Advisors provide feedback within 24-72 hours of submission. This allows enough time for in-depth edits.

More Questions

Application advising reviews.

David Flick was Incredible for ERAS!

I had a wonderful experience with David, who helped me with my residency application. He was available every step of the way, and fielded all of my questions in an extremely timely manner (often within the hour). His edits to my personal statement and CV portions were thoughtful and informed.

He also remained available throughout the interview and ranking process. He helped me with interview prep in addition to weighing in on my rank order list and how to best organize my thoughts on each program. In the end, I matched at my top choice, in no small part to David's contributions throughout the entire process. I cannot sing his praises enough!

Exceptional guidance and support for my ERAS personal statement

I am incredibly grateful for the support I received from MedSchoolCoach's Residency ERAS Personal Statement Consulting & Editing service. The team of Physician Advisors and Writing Advisors worked closely with me to understand my unique journey and craft a compelling personal statement. Their expertise in residency admissions committees was evident, and their guidance helped me highlight my strengths and aspirations effectively. With their help, I received interview invites and matched into my top-choice program. I highly recommend MedSchoolCoach for anyone seeking a competitive edge in their residency application.

Personalized attention that made a difference in my application

Working with MedSchoolCoach's Residency ERAS Personal Statement editing was an excellent decision. The personalized attention I received from the Advisors was invaluable. They took the time to understand my unique experiences and goals and guided me in developing a standout personal statement. Their expertise and insights helped me effectively convey my passion for my chosen specialty. With their help, I received interview invitations and secured a spot in my desired residency program. I am grateful to the entire team at MedSchoolCoach for their support throughout the process.

Need other help?

Great! We love working with students to help them get into medical school.

Residency Strategic Planning

We provide residency advising and early planning to get you on the right track towards your speciality choice

Residency Complete Packages

Our advisors will help you prepare a complete application for residency

Our Physician Advisors will prepare you for program interviews.

Want more comprehensive help?

Educational resources, recent blog posts.

personalized personal statement eras reddit

Medical Schools in Virginia

Check our our list of the medical schools in Virginia to find median GPA and MCAT, and tuition rates to...

August 21, 2024

Tennessee State Flag

Medical Schools in Tennessee

Check our our list of the medical schools in Tennessee to find median GPA and MCAT, and tuition rates to...

Medical Schools In Colorado

Medical Schools in Colorado

Check our our list of the medical schools in Colorado to find median GPA and MCAT, and tuition rates to...

August 20, 2024

The Pre-Med Journey

The Pre-Med Journey: What it Takes to Get into Medical School

Thinking about applying to medical school? Discover what high school students need to know about obtaining a career in medicine.

Successfully Planning for the USMLE Step 1 and 2 CK

Successfully Planning for the USMLE Step 1 and 2 CK

Get ready for the USMLE Step 1 and Step 2 with this free guide to study planning and resource utilization.

100 MCAT Study Tips

100 MCAT Study Tips

Taking the MCAT? These 100 tips and tricks will help you ace the MCAT.

Latest Reviews

Our students love working with us. Take a look at some of the latest reviews of our services.

Great experience with MedSchoolCoach!

My experience with MSC has been nothing but great so far. I started with a consultation and was not pressured into signing up right away. When I was ready, I purchased coaching, and my pre-med coach has been very helpful. I like having a physician advisor to boost my confidence about applying.

Super Helpful!!

In my BS/MD interview prep my coach thoroughly described the types of questions asked by the program, how I should go about answering them, and then gave me a mock interview. After I answered each of her interview questions, she gave me tons of feedback and told me what I should practice. Her feedback has greatly helped me prepare for my interview.

The Best Support I could Ask For

I couldn't have asked for a better college consulting service than MedSchoolCoach. Their team of advisors went above and beyond to ensure that I was well-prepared for the application process. They provided invaluable insights and helped me build a strong school list tailored to my goals. The extensive editing of my application materials helped me put my best foot forward. I'm grateful for their support and would highly recommend them to any aspiring pre-med student.

I was a 3rd time applicant to medical school…

I was a 3rd time applicant to medical school and I didn't realize how much of a difference having a great advisor could make. Medschoolcoach really made all the difference the 3rd time - was accepted to 3 MD schools, something I never thought would have been possible! I can't say enough great things about my advisors who constantly checked in and encouraged me. Would not hesitate for a second, just wish I had used them the first time!

Dr. Lee is amazing

Dr. Lee is extremely nice and down to earth. She guided my son very patiently for almost 2 years and helped him get into BS/MD program. My son has a full ride for under graduation. This entire process was daunting. But Dr. Lee and Rob Rivas helped us through the extremely stressful application and interview process. We couldn't have done this without their help and guidance. We feel blessed to have found Dr. Lee and MedSchool coach. Thanks for everything!

An invaluable resource for OMSAS applicants

The OMSAS editing service has proven to be an invaluable resource throughout my application journey. Their team of professionals provided unparalleled guidance and support. They meticulously reviewed my application, offering insightful feedback that significantly enhanced its quality.

MedSchoolCoach has been incredibly…

MedSchoolCoach has been incredibly helpful with my personal statement and secondary essays, helping me effectively communicate my experiences and qualities. They have also been a huge help with CASPer preparation. I am glad to have them assist me every step of the way with this process!

Dr. Mandalia

Dr. Mandalia provided a plethora of good ideas to work into my responses that will help me strengthen my overall narrative and presentation. He noted my strengths and weaknesses and gave me many different ways to address my weaknesses. He is clearly incredibly knowledgable about what interview committees look for and was able to elaborate on these points in great detail.

The Best MCAT Tutoring Program

MedSchoolCoach has been such a great help in my MCAT studying! My tutor, Lejla, is the best! She has helped me create my own study schedule, always makes sure I understand what we review, and provides me with MCAT-style practice questions. I wish I would have found MedSchoolCoach a lot sooner. It would have made my life so much easier. I definitely would recommend this tutoring service to my friends and peers!

Med school coach made my application process much smoother

Med school coach was extremely helpful especially developing my personal statement! They helped me put my dreams into words. It was also so wonderful to have a team of support when facing the daunting secondary essays. Highly recommend!!

Non-traditional student, exciting application process.

I'm a non-traditional student, applying for a medical residency during covid. I encountered many obstacles that had me pretty demoralized at times. Dr. Blair Nelson kept my spirits up, and demystified the process, making it seem more manageable. I'm now in orientation for my residency, and I'm not sure I'd be here without his help. We still talk and I keep him updated on my progress. This service was worth every penny. Two thumbs waaay up.

Their help got me into medical school!

As a reapplicant the advice I received got me accepted! Strengthened my personal statement, secondary essays and provided much needed assistance with interview preparation. Awesome resource to have in your corner throughout the application process.

Exceeded all expectations, seriously the best organization I could have picked

In short, this is a first class organization. They don't just take your money and let you fend for yourself. They work with you to create a personalized plan for both how much tutoring you need as well as how to most effectively use your tutoring time. Every single minute was high-yield stuff with no wasted nonsense.

If you want to succeed and are looking for the right people to help you reach your goals, look no further. Seriously.

So helpful for Step 1!

I used MedSchool Coach to help me study for the USMLE Step 1 exam and was so thankful for their help! I highly recommend Abdul was a tutor! I was really nervous about the exam and really at a loss about how to the study before working with him. He provided a personalized approach and we systematically tackled all of my areas of weakness before the exam. After our sessions, I walked into test day feeling infinitely more confident than I would have been without his help!

Call us at (888) 381-9509

Call Us Now

Or, Schedule a Meeting Below

personalized personal statement eras reddit

Get the Reddit app

/r/medicalschool is an international community for medical students

ERAS: Personalized Personal Statements vs “Love Letter” Emails after application submission

Pros vs Cons What do you all think?

By continuing, you agree to our User Agreement and acknowledge that you understand the Privacy Policy .

Enter the 6-digit code from your authenticator app

You’ve set up two-factor authentication for this account.

Enter a 6-digit backup code

Create your username and password.

Reddit is anonymous, so your username is what you’ll go by here. Choose wisely—because once you get a name, you can’t change it.

Reset your password

Enter your email address or username and we’ll send you a link to reset your password

Check your inbox

An email with a link to reset your password was sent to the email address associated with your account

Choose a Reddit account to continue

personalized personal statement eras reddit

  • Our features
  • Our Services
  • How We Work
  • Testimonials

Transitional Year Personal Statement: All You Need to Know to Get Admitted

personalized personal statement eras reddit

How Important Is Your Personal Statement for Transitional Residency Programs?

Many residencies will require you to take a transitional year program before you begin your specialized training. While in some instances this is part of the program you are applying for, in others, you will require a separate application. The need to complete a transitional year makes programs highly sought-after, and thus, the competition gets tougher.

Your transitional year personal statement will often be the most significant part of your application. You almost certainly have excellent qualifications, but they will be similar in many ways to the qualifications of those you will compete with for a place. A personal statement transitional year residency is, therefore, often your only way of showing why they should select you rather than anyone else. How to write brilliant medical residency personal statements ? Writing an effective residency application personal statement is never an easy task, no matter where you are applying. It must be done with great care in a way that is going to get you noticed. Our services have helped students with their applications for years, and we can provide you with professional advice and support to make your personal statement for a transitional year a success.

transitional year personal statement sample

What Is the Difference between Transitional, Preliminary, and Categorical Residencies?

Many international medicine programs will require you to perform a residency internship so that you can learn the ropes in several different areas before you get focused firmly on your specialty. For some programs, this will mean applying for a separate program for that internship. Different types of programs for the residency that you can apply for are:

Categorical residencies

These residency programs include a yearlong internship to gain the required skills and experience. You will apply for one residency and stay there throughout your training. Applying for a categorical year match is often the preferred route for many applicants, but not every program offers this for every specialty.

Transitional residencies

These offer a full year of training across many rotations so that you can acquire the required skills for your final specialism. These programs are also useful for those students who are still unsure of what they wish to pursue or who have failed to get directly into the specialist residency.

Preliminary residencies

These are similar to transitional residencies but will further split the training into either surgery or internal medicine preliminary years.

What Programs Require a Transitional Year?

Not every specialty requires a transitional year, so it is imperative that you know which will if you are not applying directly for a categorical residency. The following will usually require you to complete either a preliminary or transitional year as an intern prior to your specialism:

  • Anesthesiology
  • Radiation oncology
  • Dermatology
  • Diagnostic radiology
  • Physical medicine & rehabilitation
  • Ophthalmology

During your transitional year, you will typically have to cover all of the following:

  • 4 months of inpatient medicine
  • 1 month in an intensive care unit
  • 1 month of emergency medicine
  • 1 month of outpatient medicine
  • 4-5 months of electives

Get some advice on how to avoid mistakes in your medical resident resume writing!

Where Can You Apply for Your Transitional Year?

If you are applying for a transitional year, you will need to find the right one for you. The following are a selection of programs that you may consider:

  • Colorado Health Foundation Presbyterian/St. Luke’s Medical Center Program : This program is founded to offer year-long training across a wide range of different areas of clinical medicine to prepare students for their specialty.
  • Memorial Sloan Kettering Cancer Center Program: This hospital provides transitional year training in 12 departments for just 24 residents every year.
  • Scripps Mercy Hospital Program San Diego : This transitional year residency is very proud of the fact that many of those who participate in it then get placed in highly competitive residencies.
  • Presence Resurrection Medical Center Program : This program is based in Chicago. With a very busy emergency department and excellent open-heart surgery training, this is a great place to complete your year.

transitional year personal statement writing tips

How to Write Your Transitional Year Personal Statement Effectively

Crafting an effective statement that will get you noticed is a challenging task. The following tips, however, will support you in ensuring that you get yours written in a way that will give you the highest chance of success. Here are tips for a residency interview .

  • Tailor your personal statement so that it can reflect specifically what the program wants to read in it.
  • Use a transitional year personal statement sample to see how others have approached their writing: do not, however, copy anything that you have read.
  • Write about your reasons for applying to them and give them a clear idea of where their program fits into your long-term goals.
  • Do your homework and mention specific things about their program to show that your interest is serious.
  • Write about yourself!
  • Write about experiences that are irrelevant or too far in the past.
  • Use clichés within your writing: always try to be original.
  • Complain or talk negatively.
  • Incorporate language that will be difficult to understand.
  • Lie or plagiarize within your statement.
  • Fail to proofread your statement.

help with personal statement for transitional year

We Can Help You with Your Application for Transitional Residency Programs

Whether you are writing a future long-term plan essay for the medical school or a transitional residency personal statement, our services can help you. We provide expert support through fully qualified staff that know precisely what the admissions committee will be looking for. All of our writing support will always result in a piece of writing perfectly tailored to your needs.

We always provide our help on time, and every document is delivered with a free plagiarism report. With a full satisfaction or money-back guarantee, there is no reason not to use our service.

transitional year personal statement writing service

Contact our highly skilled writers here today to ensure that your transitional year personal statement will get you noticed.

nurse residency programs in california

Nurse Residency Programs in California: A Comparison List

Are you looking at nurse residency programs in California? If so, you need to do research into the different residencies before you select one. This way, you’ll make certain that the program you’ve selected is the one that can offer you the most. You want to be sure you’re going to be a good fit […]

Preparing Your CV for VSAS

ERAS Personal Statement Word Count

As the most significant part of your application, an ERAS personal statement can be your greatest chance to succeed. Basically, this type of document will help you to state the reason why you’ve chosen that particular field and convince the committee that you’re the best candidate for the position. Whereas you don’t have control over […]

  • Communities Pre-Med Medical Resident Audiology Dental Optometry Pharmacy Physical Therapy Podiatry Psychology Rehab Sci Veterinary
  • What's new Trending New posts Latest activity
  • Support Account Help Confidential Advising
  • Vision, Values and Policies
  • Medical Student Communities
  • Medical Students (MD)

Residency application - individual personal statement for every program

  • Thread starter stefan95
  • Start date Aug 16, 2021

personalized personal statement eras reddit

  • Aug 16, 2021

hiiiiiiiiiii

Full member.

stefan95 said: Hi everyone, I am an IMG applying to IM residency for the upcoming season. I was just wondering: Will you need an individual personal statement for every program that you apply to? Or is it common to just use the same personal statment for all your applications within one specialty, considering that many applicants apply to >100 programs? Many thanks in advance. Stefan Click to expand...

Dral

I personalized mine for my top few. Overall, just use the same one for all places though.  

Similar threads

KaioKaizen

  • Sep 13, 2023

Chioma Nwabuoku

  • Chioma Nwabuoku
  • Jul 7, 2024
  • Residency Program Specific Discussions

itsalwayssunny96

  • itsalwayssunny96
  • Jun 25, 2022
  • happybay_premed
  • Jun 18, 2023

Angler

  • Aug 13, 2023

Rachapkis

  • This site uses cookies to help personalize content, tailor your experience and to keep you logged in if you register. By continuing to use this site, you are consenting to our use of cookies and terms of service . Accept Learn more…

IMAGES

  1. How to write ERAS residency personal statement

    personalized personal statement eras reddit

  2. How to Prepare ERAS Personal Statement Properly

    personalized personal statement eras reddit

  3. eras personal statement examples neurology

    personalized personal statement eras reddit

  4. ERAS Personal Statement Components

    personalized personal statement eras reddit

  5. How to write a personal statement -reddit wiki

    personalized personal statement eras reddit

  6. ERAS Personal Statement Length

    personalized personal statement eras reddit

VIDEO

  1. How To Write Personalize Personal Statement or Statement of Purpose using ChatGPT & Gemini

  2. My sister 18 is dating a man in his 40's.....with updates!!

  3. Medical Personnel, what was the most uneducated statement a patient has said to you Best Reddit

  4. How to create a personalized and personal ERAS application

  5. Red Flags on Residency Application #match2025

  6. FutureDocs Podcast

COMMENTS

  1. [Residency] Personalizing the Personal Statement for each ...

    Most personal statement examples that I've consulted online suggest including a section like: "how I would be a good fit for your program." I plan on having 3/4 of my statement being standardized, with some space for personalization in the final paragraph, i.e. why I'm interested in the program. I also will add a bit more at the end for the ...

  2. Current fourth years through PGY-2s: Did you personalize your ...

    Every stage of becoming a physician is an arms race at this point. In 5 years it will be fully personalized PS's with away rotations, phone calls from your PD to theirs, and 4 letters all in the specialty you are applying to with 4 letters in your backup specialty.

  3. How to Write a Killer Personal Statement for [Residency] Apps ...

    As promised, here is the "idiot-proof" template: Paragraph 1. Start with a compelling hook. Set the mood with a vivid anecdote of you engrossed in something you care about. Bonus points if this features a hobby that's not directly tied to medicine.

  4. Worth the effort to personalize your personal statement to every

    Jul 26, 2005. #1. It wouldnt be that much trouble, I suppose, but would it be advantageous to insert something personal in the statement that relates to each program, since you can assign different PS to each one. Like, this is why I would be a good fit at _________ U. medical center, etc. Not necessarily that blatant, but something along those ...

  5. Customizing personal statements in ERAS

    Nov 8, 2011. #1. Most people use multiple personal statements because they are applying to multiple specialties, but thats not the case with me. I'm only applying to one fellowship subspecialty, probably just a few programs, and I'm wondering how useful it would be to create customized personal statements for each program, especially my top 1-3 ...

  6. Personal Statement

    The personal statement is limited to 28,000 characters, which include letters, numbers, spaces, and punctuation marks. There is not a limit to how many personal statements applicants can create. Personal statements created outside the MyERAS application should be done in a plain text word processing application such as Notepad (for Windows ...

  7. Residency Personal Statement: The Ultimate Guide (Example Included)

    The personal statement is an essay of about a page (one page in ERAS is 3,500 characters including spaces) in which you articulate who you are and why you want to enter a certain specialty. It's your big opportunity to set yourself apart from other applicants by highlighting anything that isn't well represented in other parts of your ...

  8. Does a 'personalized' personal statement help?

    15+ Year Member. Jul 15, 2008. #2. You know, a few years ago I would have said it doesn't matter much. However, I did do this (personalize some of the personal statements) while applying to cards fellowships this year, and it seems to have worked. I'd say if you are within the range of applicants they accept, then it might help you.

  9. 12 Top Questions About the ERAS Personal Statement

    78% of program directors cite the ERAS personal statement as an important factor in deciding which candidates to interview, making it the fourth-highest ranked factor behind USMLE Step 1, USMLE Step 2, and letters of recommendation. So, it's pretty important in the grand scheme of your application!

  10. How to structure the ERAS personal statement? : r/medicalschool

    Go with whatever idea first pops into your brain and run with it, try not to edit yourself as you go. One thought will lead to another and eventually you'll have at least something to grow from. The app tells programs about your accomplishments, hobbies, other people's perspective of you.. but, it doesn't give them a sense for your personality ...

  11. Writing an Impressive Residency Personal Statement

    Ensure your personal statement flows well; The best personal statements are easy to read, don't make the reader think too much, and make your path and interests seem logical. Rarely does a personal statement have a theme. Also try to have each paragraph transition to the next seamlessly. 2. Your personal statement should be about you!

  12. Residency Personal Statement Examples from Matched Residents

    Residency Personal Statement Examples #6: Cardiology. "Code blue, electrophysiology laboratory" a voice announces overhead during my cardiology rotation. As the code team, we rush to the patient, an elderly man in shock. Seamlessly, we each assume our preassigned roles.

  13. Personal statement editing in ERAS

    Hello all! I have a question about how to enter the personal statement in ERAS. I have a Macbook Air, and I typed my statement in Microsoft Word, but when I try to copy-and-paste the statement from Word to a simple text editor (TextEdit), and then copy-and-paste the statement from TextEdit to ERAS, my 750-word statement looks like only half a page.

  14. Can you tailor your personal statement to each Residency program?

    yes. I don't think it's necessary. It is definitely feasible, in theory you can upload as many personal statements as you want to ERAS and assign each program a unique statement. I can't comment on how a gen surg program would view a personal statement specific to them.

  15. Documents for ERAS® Residency Applicants

    Personal Statement. The personal statement may be used to personalize the application to a specific program or to different specialties. There is not a limit to how many personal statements you may create; however, you may only assign one (1) for each program. Note: There are a number of websites that provide examples of Personal Statements. Do ...

  16. Personalized personal statement : r/medicalschool

    More and more FM programs are requiring it. Applying to Gas. All 5 gold signals are getting personalized PS. 5/10 silver signals getting personalized PS. For my signals only. personalizing for my gold signals for anesthesia (5) and then also for a program in my home state. Slightly different for prelims vs residencies.

  17. For ERAS do we write a personal statement for each specific program?

    Going overboard and writing a new PS for each institution you're applying to is probably overboard. The multiple PS scenario is typically for people that are applying to multiple specialties. 1 user. Jul 31, 2019. #5. If you do write multiple statements, be *VERY* careful that you assign the correct PS to each program.

  18. Residency ERAS Personal Statement Consulting & Editing

    Residency Personal Statement Editing. $650. -. Sign Up. Personalized Attention with a Residency Physician Advisor to Develop Your Concepts & Finalize Your Submission. Up to 3 Rounds of Grammar, Style and Content Editing with a Professional Writer. Video & Resources to Guide the Development of Your Essay. 30 minutes Brainstorming Time.

  19. ERAS: Personalized Personal Statements vs "Love Letter ...

    Realistically , they don't carefully read the personal statement of every applicant. I'd also do a love letter email if you're trying to unlock an interview, and/or if you're trying to confirm your interest after the interview process.

  20. Transitional Year Personal Statement to Get You Noticed

    Here are tips for a residency interview. Always: Tailor your personal statement so that it can reflect specifically what the program wants to read in it. Use a transitional year personal statement sample to see how others have approached their writing: do not, however, copy anything that you have read. Write about your reasons for applying to ...

  21. Individual Personal Statements For Different Programs

    Some of my friends in med school right now applying for residency are writing multiple personalized personal statements tailored to different programs. Is this something also expected / typical for OMFS? ... Facebook X (Twitter) Reddit Pinterest Tumblr WhatsApp E-mail Share Link. Next unread thread Similar threads. R. Personal Statement ...

  22. Program specific personal statement

    That said, you could do something like have 2-3 personal statements. For ex, perhaps your personal statement is very research focused, but you don't want to give that impression to your community program backups. It's reasonable to send them an alternative personal statement that doesn't have that research focus. What I wouldn't do is have a ...

  23. Residency application

    Just use the same PS for every program. Unless there is one in particular you really want to match to then you could potentially personalize that one slightly but def not necessary. 1 user. Aug 16, 2021. #3. I personalized mine for my top few. Overall, just use the same one for all places though.